You are on page 1of 103

Practice Test in General Education Mathematics-LET

1. What is the missing term in the series? 2, 5, 11, 23, 47, ____
a. 115 b. 95 c. 105 d. 125
2. A store owner makes a 25% profit by selling an item for P800.00. How much is his profit?
a. P 1,400.00 b. P 200.00 c. P 250.00 d. P 160.00
3. The simplest expressions for 25/100 is ______
a. 1 b. 4 c. d. 1/4
4. If the ratio of women to men in a meeting is 4 to 1, what percent of the persons in the
meeting are men?
a. 80% b. 20% c. 33 1/3 % d. 25%
5. Joel repacks a 60 kg sack of sugar into small packs of 750 g. How many small packs can be
made?
a. 90 b. 45 c. 55 d. 80
6. How much bigger is 550 than 375?
a. 175 b. 73 c. 45 d. 431
8. The product of 2 numbers is 36, and their ratio is 1: 4. Which of these is the smaller
number?
a. 9 b. 12 c. 3 d. 2
9. An Arithmetic book is 2 cm thick. How many copies can be placed in a meter long shelf?
a. 44 b. 40 c. 36 d. 45
10. What is the smallest positive number that is a multiple of both 12 and 14?
a. 36 b. 42 c. 84 d. 168
11. 40% of 35 is what percent of 140?
a. 28% b. 29% c. 10% d. 14%
12. An employee earning P 9,200.00 a month will receive 15% increase next month. How much
will his/her new salary be?
a. P10,500 b. P10,530 c. P10,580 d. 10,560
13. In a certain school, the ratio of boys to girls is 3 to 7. If there are 150 boys and girls in the
school, how many boys are there?
a. 45 b. 90 c. 105 d. 75
14. A park has a triangular shape. It has a base of 12m and a height of 9m. What is the area?
a. 122 sq. m b. 42 sq. m. c. 108 sq. m. d. 54 sq. m.
15. The length of a new piece of chalk is about _________.
a. 1mm b. 10mm c. 10cm d. 1 cm
16. If 50% of x is 20, what is 30% of x?
a. 30 b. 12 c. 20 d. 16
17. Which of these fractions has the greatest value?
a. 4/5 b. 9/16 c. 7/10 d. 5/8

18. Which of the following is exactly divisible by 3 and 11?


a. 357404 b. 114345 c. 991111 d. 135792
19. If the variance of distribution is 25, then the standard deviation is _________.
a. 30 b. 5 c. 625 d. 12.5
20. Which of these numbers is greater than ?
a. 0.04 b.0.09 c. 1/8 d. 1/0.04

23. A senior class of 50 girls and 70 boys sponsored a dance. If 40% of the girls and 50% of the
boys attended the dance, approximately what percent attended?
a. 44 b. 46 c. 42 d. 40
24. What is the smallest positive integer that has 6, 8, and 10 as factors?
a. 300 b. 240 c. 80 d. 120
25. Joseph has more money than Mila but less than Billy. If the amounts held by Joseph, Mila and
Billy are x, y, and z respectively, which of the following is TRUE?
a. Z<X<Y b. X<Z<Y c. Y<X<Z d. Y<Z<X
26. How many ounces of pure acid must be added to 20 ounces of a solution that is 5% acid in
strength?
a. 2 b. 1 c. 3 d. 5

28. Michael is 15 years older than this brother Rafael. However, y years ago Michael was twice as
old as Rafael. If Rafael is now x years old and x>y, find the value of x y.
a. 13 b. 14 c. 15 d. 16
29. What percent is of ?
a. 65% b. 90% c. 80% d. 75%
30. What is the median of the following numbers? 8,5,7,5,9,9,1,8,10,5 and 10.
a. 9 b. 8 c. 5 d. 7

31. A certain pole casts a shadow 24 feet long. At the same time another pole 3 feet high casts a
shadow 4 feet long. How high is the first pole, given that the heights and shadows are in
proportion?
a. 20 ft. b. 24 ft. c. 18 ft. d. 21 ft.

33. The perimeter of an isosceles triangle is 42 in. The 2 equal sides are each 3 times as long as
the third side. What are the lengths of the three sides?
a. 18, 21, 3 c. 21, 21, 21
b. 6, 6, 8 d. 18, 18, 6

36. If a certain job can be finished by 18 workers in 26 days, how many workers are needed to
finish the job in 12 days?
a. 45 b. 39 c. 24 d. 30

38. A box is 12 inches wide, 16 inches long, and 6 inches high. How many square inches of paper
would be needed to cover it on all sides?
a. 192 b. 720 c. 360 d. 900

39. P 8,000.00 is invested at 12% simple interest. What is the amount at the end of 2 years?
a. P 9,600 b. P 8,960 c. P 8,240 d. P9,920

Answer Key
1. B
2. B
3. D
4.B
5.D
6.A
8. C
9. B
10. C
11. C
12. C
13. A
14. D
15. C
16. B
17. A
18. B
19. B
20. D
23. B
24. D
25. C
26. B
28. C
29. D
30. B
31. C
33. D
36. B
38. A
39. D
LET ENGLISH LITERATURE PRACTICE TEST

1. Who wrote this line? Where ignorance is bliss, it is folly to be wise.


a. Robert Browning
b. William Shakespeare
c. Rudyard Kipling
d. Edgar Allan Poe

2. What nationality was Robert Louis Stevenson, writer of Treasure Island?


a. English
b. Welsh
c. Irish
d. Scottish

3. Which Bronte writer authored Jane Eyre?


a. Charlotte
b. Emily
c. Cristina
d. Anne

4. In which century were Geoffrey Chaucers Canterbury Tales written?


a. 14th
b. 15th
c. 16th
d. 17th

5. The following taboo phrases were used by which writer? I fart at thee, shit on your head,
dirty bastard
a. Ernest Hemingway
b. Henry James
c. Ben Johnson
d. Arnold Bronte

6. In the book The Lord of the Rings, who or what is Bilbo Baggins?
a. man
b. hobbit
c. wizard
d. dwarf

7. Name the book which opens with the line All children, except one grew up?
a. The Jungle Book
b. Tom Sawyer
c. Peter Pan
d. The Adventures of Huckleberry Finn
8. How many lines does a sonnet have?
a. 12
b. 13
c. 14
d. 15

9. Who was the author of the famous storybook Alices Adventures in Wonderland?
a. H.G. Wells
b. Lewis Carroll
c. Mark Twain
d. E.B. White

10. Cabbages and Kings (1904) is either a novel or a collection of related short stories written
by O. Henry. In it, he coined the phrase banana republic. On what was his title based?
a. Mark Twains The Prince and the Pauper
b. Alice Hegan Rices Mrs. Wiggs of the Cabbage Patch
c. The Shahnameh an 11th Century Persian epic poem
d. Lewis Carrolls poem The Walrus and the Carpenter

11. Two versions of Robert A. Heinleins novel Stranger in a Strange Land have been publishe
d:
the edited version first published in 1961 and the original fulllength (60,000 words longer) publ
ished posthumously in 1991. From what does the title derive?
a. The play Antony and Cleopatra by William Shakespeare
b. The Old Testament Book of Exodus
c. The novel Gullivers Travels by Jonathan Swift
d. The book Utopia by Sir Thomas More

12. Southern American poet, novelist and literary critic Robert Penn Warren wrote All the King
s Men in 1946. The novel won the 1947 Pulitzer Prize for Fiction. On what is the books title ba
sed?
a. A verse in the nursery rhyme Humpty Dumpty
b. William Shakespeares play Richard III
c. Oscar Wildes short story The Young King
d. Joyce Kilmers poem Kings

13. Which novel, eventually published in 1945, was rejected by a New York publisher stating it i
s impossible to sell animal stories in the USA?
a. Animal Farm
b. Black Beauty
c. Watership Down
d. The Tale of Peter Rabbit

14. Which writer of spy fiction, and creator of Smiley, was rejected with the words you are welc
ome to **** he hasnt got any future?
a. Ian Fleming
b. John le Carr
c. Eric Ambler
d. Len Deighton

15. The Good Earth was rejected fourteen times, before being published and going on to win t
he Pulitzer Prize. Who was the author?
a. Pearl S. Buck
b. John Steinbeck
c. Edith Wharton
d. Henry Miller
16. Irving Stones Lust for Life was rejected sixteen times, with one rejection stating a long, d
ull, novel about an artist. Which artist did the book feature?
a. Sigmund Freud
b. John Noble
c. Michelangelo
d. Vincent Van Gogh

17. Who is presented as the most honest and moral of Chaucers pilgrims?
a. The Knight
b. The Parson
c. The Reeve
d. The Wife of Bath
18. Out of the following four pilgrims, which is the most corrupt?
a. The Sergeant /Man of Law
b. The Wife of Bath
c. The Reeve
d. The Pardoner

19. He translated The Fall of Princes from the French.


a. William Langland
b. Sir Thomas Malory
c. Geoffrey of Monmouth
d. John Lydgate

20. What work contains these lines: There hurls in at the hall-
door an unknown rider . . . Half a giant on earth I hold him to be.
a. Sir Gawain and the Green Knight
b. Morte Darthur
c. Piers Plowman
d. Canterbury Tales

ANSWER KEY and EXPLANATION for ENGLISH LET PRACTICE TEST


1. B William Shakespeare
2. D Scottish
Robert Louis Balfour Stevenson was a Scottish novelist, poet, essayist and travel writer. His bes
t-known books include Treasure Island, Kidnapped, and Strange Case of Dr Jekyll and Mr Hyde.
3. A Charlotte
Charlottes Jane Eyre was the first to know success, while Emilys Wuthering Heights, Annes T
he Tenant of Wildfell Hall and other works were later to be accepted as masterpieces of literatur
e. Christina Georgina Rossetti was an English poet who wrote a variety of romantic, devotional,
and childrens poems. She is best known for her long poem Goblin Market, her love poem Reme
mber, and for the words of the Christmas carol In the Bleak Midwinter.
4. A 14th
The Canterbury Tales is a collection of stories written in Middle English by Geoffrey Chaucer at
the end of the 14th century.
5. C Ben Johnson
6. B hobbit
Bilbo Baggins is the protagonist and titular character of The Hobbit and a supporting character
in The Lord of the Rings, two of the most well-known of J. R. R. Tolkiens fantasy writings.
7. C Peter Pan
Peter Pan is a character created by Scottish novelist and playwright J. M. Barrie (1860
1937). A mischievous boy who can fly and magically refuses to grow up, Peter Pan spends his n
ever-
ending childhood adventuring on the small island of Neverland as the leader of his gang the Los
t Boys, interacting with mermaids, Indians, fairies, pirates, and (from time to time) meeting ordi
nary children from the world outside.
8. C 14
The term sonnet derives from the Occitan word sonet and the Italian word sonetto, both mea
ning little song or little sound. By the thirteenth century, it had come to signify a poem of fo
urteen lines that follows a strict rhyme scheme and specific structure.
9. B Lewis Carroll
Some of H.G. Wells works are The Time Machine, The Island of Doctor Moreau, The Invisi
ble Man, The War of the Worlds. He is also known as the Father of Science Fiction. Mark Twa
in is most popular in his Tom Sawyer and The Adventures of Huckleberry Finn. E.B. White is
well known of her novel Charlottes Web.
10. D Lewis Carrolls poem The Walrus and the Carpenter
11. B The Old Testament Book of Exodus
Moses fled Egypt and married Zipporah. And she bare him a son, and he called his name Gers
hom: for he said, I have been a stranger in a strange land. Exodus 2:22 Authorized (King Jame
s) Version.
12. A A verse in the nursery rhyme Humpty Dumpty
Robert Penn Warren is the only person to have won Pulitzer Prizes for both fiction and poetry.
A commemorative postage stamp was issued in the United States in 2005 to honor the 100th an
niversary of his birth. Stage plays, television versions, several movies and even a grand opera h
ave been based on Warrens novel.
13. A
Animal Farm was written by George Orwell, and is a satire on revolution and the corruption of
power. One of the best known lines from it is all animals are equal, but some animals are more
equal than others. The rejection notice implies that the publisher did not actually read the boo
k or totally misunderstood it if he did. Watership Down was written by Richard Adams and publ
ished in 1972. Anna Sewell wrote Black Beauty, which appeared in 1877 and Beatrix Potter wa
s the author of The Tale of Peter Rabbit from 1902.
14. B John le Carr
This was a rejection notice for The Spy Who Came in From the Cold, which found another pub
lisher in 1963. Le Carr had worked for both MI5 and MI6, the British intelligence services, and l
eft to become an author full time following the success of this novel. Among Len Deightons nov
els are The Ipcress File and Eric Ambler wrote The Mask of Dimitrios. Fleming, of course, is th
e creator of probably the most famous spy of all in James Bond.
15. A Pearl S. Buck
One rejection notice read I regret that the American public is not interested in anything on Chi
na. The novel was published in 1931 and won the Pulitzer Prize the following year. Pearl S Buck
wrote numerous other novels, including East Wind, West Wind, short stories, biographies and
non-fiction works and won the Nobel Prize for Literature in 1938.
16. D Vincent Van Gogh
The book was published in 1934 and was so successful that it was made into a film of the sam
e name, starring Kirk Douglas, in 1956. Irving Stone also wrote about all the other names given
as options. Michelangelo was the subject of The Agony and the Ecstasy, published in 1961 an
d also filmed, with Charlton Heston, in 1965. John Noble, an American artist, was the subject of
The Passionate Journey from 1949. Sigmund Freud, the psychoanalyst, was covered in The P
assions of the Mind in 1971.
17. B The Parson
Despite the immorality that is apparent amongst the clergy, hope manifests itself in the form of
the Parson, who is presented as an almost Christ-
like figure. Although materially poor, he is spiritually empowered, for riche he was of both ho
oly thoght and werk. Yet for every trap that Chaucers Parson has avoided, there are thousand
s that have fallen into them, and in light of this, the goodness of Chaucers Parson only serves t
o heighten the unruliness that is present in everybody else. For in the General Prologue he is t
he only individual that completely measures up to the strict Christian ideal, which is something
even the Church itself does not.
18. D The Pardoner
The Pardoner, is certainly presented as one of the most corrupt of all Chaucers pilgrims (along
with the Summoner), making both the person and the peple his apes. His deception and fey
ned flaterye convinces simple folks to purchase his phoney relics. He cheats and manipulates al
l that believe in the sanctity of the Church and the morality of those that represent it, so much s
o, that Chaucer himself can find nothing good to say about him. For thought He was in chirche
a noble ecclesiaste, this is merely an act, for he would preche, and wel affile his tonge for th
e sole purpose of of winning silver from the crowd.
19. D
He also translated The Siege of Thebes. The Fall of Princes is based on another work by Bo
ccaccio. Lydgate is little known today, but in his own time he was nearly as renowned as Chauc
er.
20. A Sir Gawain and the Green Knight
The author of this Arthurian tale is unknown, but he is thought to have also written the poems
Patience, Pearl, and Purity.
LET Practice Test in Physical Science

MULTIPLE CHOICE.
1. Most cars have four-stroke internal combustion engines. Which of the following gives the
correct sequence of the four strokes?
A. Power-compression-intake-exhaust
B. Compression-exhaust-power-intake
C. Intake-power-compression-exhaust
D. Intake-compression-power-exhaust
2. The image in a plane mirror is _______________.
A. Inverted, virtual with a magnification of 1
B. upright, real with a magnification of 2
C. inverted, real with a magnification of
D. upright, virtual with a magnification of 1
3. Which of the following wave characteristics remains unchanged when a wave crosses a
boundary into a different medium?
A. Velocity C. Frequency
B. Amplitude D. Wavelength
4. Which of the following electromagnetic radiations has the lowest frequencies?
A. Gamma rays C. Ultraviolet rays
B. Radio waves D. X-rays
5. If something gets a positive electric charge, then follows that something else__________.
A. become magnetized
B. becomes equally negatively charged
C. becomes equally positively charged
D. becomes negatively charged, but not necessarily equally negatively charged
6. Hydrocarbons which do NOT have maximum number of hydrogen atoms are _______.
A. aromatic C. saturated
B. isometric D. unsaturated
7. Metals have been used for making coins and jewelry. Which of the following characteristics
is true of metals?
A. Metals are malleable and ductile
B. Metals have high tensile strength
C. Metals are good conductors of heat and electricity
D. all of the above
8. Which of the following is that single force applied at the same point that will produce the
same effect?
A. reactant force C. resultant force
B. composite force D. concurrent force
9. How many grams of chloral are needed to produce 10.5 g of DDT?
A. 10.5 g C. 4.36 g
B. 8.72 g D. 13.1 g
10. The half-life Carbon- 14 is 6739 years. Suppose a container was found to contain one half
as much Carbon- 14 as a new basket of the same material. What is the approximate age of the
bowl?
A. 3716 years C. 2856 years
B. 1432 years D. 6739 years
11. What kind of energy is present whenever a body is at a distance from the ground?
A. elastic potential energy
B. electric potential energy
C. electromagnetic potential energy
D. gravitational potential energy
12. A lunar eclipse occurs when the ____________.
A. moon is between the sun and the earth
B. moon, earth and sun form a right angle
C. earth is between the moon and sun
D. sun is in between moon and earth
13. How long does it takes the moon to go through all of its phases?
A. 1 year B. 29 days C. 24 hours D. 27 1/3 days
14. Which of these minerals is present in seawater with the highest percentage?
A. Sodium chloride C. Magnesium chloride
B. Magnesium sulfate D. Calcium sulfate
15. What must be the minimum length of a plane mirror in order for you to see a full view o f
yourself?
A. Your full height C. One-half
B. One-forth D. Three-fourths
16. At what particular instance will the period and frequency of a wave changed?
A. When it travels from one medium to another
B. When it is at its equilibrium position
C. When it interferes with another wave
D. When it reflects
17. What laws states that there is decrease in pressure at the side of a moving stream of fluid?
A. Bernoullis Law C. Archimedes Law
B. Pascals Law D. Hookes Law
18. Whose discovery involved the production of magnetism by means of electricity?
A. Ohm B. Lenz C. Faraday D. Oersted
19. In the absence of air friction, what happens when a balls allowed to roll down one hill and
up a second hill?
A. The ball would lose speed more slowly as it moves up the second hill.
B. The ball would travel a longer distance uphill.
C. The ball would travel a shorter distance as it moves up.
D. The ball would practically reach the same horizontal level from where it started.
20. A car travels at 80 km/hr for 0.5 hr, then goes 100 km/hr for 1 hour, and finally 60 km/hr
for 0.5 hr. What is the average speed for the 2-hour travel?
A. 80 km/hr C. 100 km/hr
B. 90 km/hr D. 85 km/hr
21. In Mohs scale, which is the softest and the hardest mineral?
A. Talc Diamond C. Calcite Feldspar
B. Gypsum Quartz D. Talc Corundum
22. According to the kinetic molecular theory, liquids are similar to solids in that ___________
A. their molecules are arranged in a regular manner.
B. the motion of their particles changes with temperature.
C. the motion of their particles cannot be determined.
D. there is very little space between their molecules.
23. When two or more objects collide the quantity that does NOT change is the total
__________.
A. Momentum of all the objects C. kinetic energy of all the
objects
B. kinetic energy of each object D. momentum of each object
24. What is the correct formula for finding the efficiency of a machine?
A. Efficiency = input force /output force x 100%
B. Efficiency = output force/input force x 100%
C. Efficiency = output work/input work x 100%
D. Efficiency = input work/ output work x 100%
25. The energy of a hammer hitting a nail is
A. heat C. electric
B. chemical D. mechanical
26. Disregarding air resistance, an object falling towards the earths surface has an acceleration
that is _____________.
A. decreasing C. constant
B. increasing D. depending on objects
weight
27. If the moons gravity becomes 1/4 that of Earth, which of the following will apply?
I. The moon will no longer cause tides on the earths oceans.
II. A 65kilo person on earth will weigh about 18 kilos on the moon.
III. It will be easy to lift heavy objects on the moon.
IV. The moon will not hold a thick atmosphere of gasses.
A. II, III and IV C. I, II and III
B. I, II and IV D. I, II, III and IV
28. The headings of a car with a 12V electrical system are 40W each the tail lights 6.0 W each,
and the radios is 20 W. What is the total power of this circuit?
A. 1200 W C. 110 W
B. 240 W D. 3600 W
29. Abundance of waste materials in water decreases primary productivity due to ___________.
A. blockage of light penetration
B. amount of nutrients available
C. reduced amount of oxygen in the water
D. accumulation of aquatic organisms
30. What is the direction of the magnetic field in an electromagnetic wave?
A. parallel to the electric field C. parallel to the motion
B. perpendicular to the electric field D. random
31. A ball of mass 0.15 is thrown with a speed of 20 m/s against a wall. What is its momentum
if it rebounds after colliding elastically?
A. +3.0 kg m/s B. -30 kg m/s C. -3.0 kg m/s D. +30 kg
m/s
32. How are echoes minimized in halls and auditoriums?
A. covering hard reflecting surfaces with curtains
B. opening the doors of the auditoriums
C. creating a wider space for sound to travel
D. limiting the accessories installed inside
33. A large depression caused by a collapse of the slopes of a volcano is called a _________.
A. basin C. base level
B. syncline D. caldera
34. Where does the pick up needle on a phonograph move the fastest across the record?
A. At the end
B. The needle move at the same speed throughout
C. In the middle of the phonograph record
D. At the beginning
35. Compared to the density of seawater, a kind of fish that lives at the bottom of the ocean
has a density that is __________.
A. lesser C. same
B. greater D. equal
36. The additional of Br2 is used as the reaction to distinguish between alkenes and alkanes.
Which of the following observations accompanies this test?
A. When Br2 is added to the double bond of alkenes to make a dibromide, it
becomes colorless.
B. A bright red color is produced when Br2 reacts with alkenes.
C. The red color of bromine disappears when it dissolves in alkenes.
D. Bromine dissolved in alkenes but not in alkenes.
37. Which statement is CORRECT about a rechargeable battery?
A. Its REDOX reactions are not complete.
B. Its REDOX reactions are not reversible.
C. Its REDOX reactions are complete.
D. Its REDOX reactions are reversible.
38. A ray of light is incident on a plane surface separating two transparent substances of
refractive indices 1.70 and 1.50. The angle of incidence is 30 deg and the ray originates in the
medium of higher index. The angle of refraction is _________.
A. 79.0 C. 45.0
B. 34.5 D. 26.2
39. Chrome plating is one method to prevent corrosion. This process can be carried out
by_______.
A. using secondary cells C. using primary cells
B. electrolytic process D. zone refining
40. What does the slope of the graph of stretching force against elongation indicate?
A. elastic limit C. gravitational limit
B. resistivity constant D. spring constant

GOOD LUCK! Prof. Cris Paner

Answer Key:
1. D 11. D 21. A 31. C

2. D 12. C 22.B 32. A


3. C 13. B 23. A 33. D
4. B 14. A 24. C 34. B
5. D 15. C 25. D 35. D
6. D 16. C 26. C 36. A
7. D 17. A 27. C 37. D
8. D 18. C 28. A 38. A
9. D 19. A 29. C 39. B
10. D 20. D 30. B 40. D
LET Practice Test 1 General Science

1.Which is a destructive fishing method wherein fishes are driven out of a coral reef by
pounding the corals with heavy weight?

A. Corals entrapping

B. Reef fishing

C. Cyanide fishing

D. Muro-ami

2. What may occur when there is a severe damage to some parts of the brain due to lack of
blood supply?

A. Heart attack

B. Rheumatic heart

C. Stroke

D. Thrombosis

3. Which can help prevent developing osteoporosis?

A. Adequate intake of calcium and vitamin D

B. Less exercise to decrease bone density

C. Adequate intake of Vitamins A and D

D. Estrogen replacement therapy

4. Meteorology is a very important branch of natural science, without which we cannot be


informed of forthcoming _______.

A. earthquakes
B. typhoons

C. comets

D. meteors

5. In the energy pyramid, which organisms occupy the base?

A. Herbivores

B. Heterotrophs

C. First order consumers

D. Producers

6. One approach to lessen vehicular air pollution is:

A. remove twenty or more year old vehicles in the street

B. ban diesel using cars on the highway

C. ask pedestrians to use bicycles

D. utilizing the MRT and LRT

7. The theory on the effective use of cellular phones is credited to _______?

A. Smart

B. McDonald

C. Bell

D. PLDT

8. Empty bottles and cases of pesticides must be disposed of properly. This is done this way:

A. burying containers in deep pits

B. washing with soap suds

C. burning in a pit

D. breaking the bottles in waste boxes

9. Environmentalists believe that___________.

A. opening dumping sites will solve waste disposal best.

B. burying plastic will ease clogging of water canals.

C. classifying garbage identify biodegradable and non-biodegradable waste


D. burning garbage is the safest approach.

10. Famine in Africa found very malnourished children. Several had poor teeth and sight. This is
a sign of ___________________.

A. deficiency in Vitamin A and D

B. deficiency in protein

C. deficiency in Vitamin B

D. deficiency in iron

11. If something gets a positive electric charge, then follows that something else__________.

A. become magnetized

B. becomes equally negatively charged

C. becomes equally positively charged

D. becomes negatively charged, but not necessarily equally negatively charged

12. A lunar eclipse occurs when the ____________.

A. moon is between the sun and the earth

B. moon, earth and sun form a right angle

C. earth is between the moon and sun

D. sun is in between moon and earth

13. In Mohs scale, which is the softest and the hardest mineral?

A. Talc Diamond C. Calcite Feldspar

B. Gypsum Quartz D. Talc Corundum

14. The energy of a hammer hitting a nail is

A. heat C. electrical

B. chemical D. mechanical

15. A large depression caused by a collapse of the slopes of a volcano is called a _________.

A. basin C. base level

B. syncline D. caldera

Answer Key:
1. D

2. C
3. A
4. B
5. D
6. C
7. C
8. A
9. C
10. A
11. D
12. C
13. A
14. D
15. D
LET PRACTICE TEST 1- PROFESSIONAL EDUCATION

1. Which violate(s) the principle of respect?


I. Teacher A tells her students that what Teacher B taught is wrong.
II. To retaliate, Teacher B advises students not to enroll in Teacher As class.
III. Teacher C secretly gives way to a special favor (e.g. add 2 points to grade) requested
by student A who is vying for honors.
A. II and III
B. I, II and III
C. I and II
D. I and III
2. Which is/are in accordance with the principle of pedagogical competence?
I. Communication of objectives of the course to students
II. Awareness of alternative instruction strategies
III. Selection of appropriate methods of instruction
A. I and III
B. I, II and III
C. III only
D. II and III
3. With a death threat over his head, Teacher D is directed to pass an undeserving student.
Which will a utilitarianist do?
A. Pass the student, why suffer the threat?
B. Dont pass him; live by your principle of justice. You will get reward, if not in this life, in the
next.
C. Pass the student. That will be of use to the student, his parents and you.
D. Dont pass him. You surely will not like someone to give you a death threat in other to pass.
4. Teacher A knows of the illegal activities of a neighbor but keeps quiet in order not to be
involved in any investigation. Which foundational principle of morality does Teacher A fail to
apply?
A. The end does not justify the means
B. The end justifies the means
C. Always do what is right
D. Between two evils, do the lesser evil
5. To earn units for promotion, a teacher pays her fee but does not attend class at all. Does this
constitute professional growth?
A. Not immediately but yes after promotion
B. It depends on the school she is enrolled in
C. No, it is simply earning MA units for promotion
D. Yes, just enrolling in an MA program is already professional growth
6. If a teacher asks more higher-order questions, he has to ask more _______ questions.
A. fact
B. close
C. convergent
D. concept
7. Misdemeanor has a ripple effect. This implies that as a classroom manager, a teacher
___________.
A. reinforces positive behavior
B. responds to misbehavior promptly
C. is consistent in her classroom management practice
D. count 1 to 10 before she deals with a misbehaving student
8. Based on Edgar Dales Cone of Experience, which activity is farthest from the real thing?
A. Watching demo
B. Attending exhibit
C. Video disc
D. Viewing images
9. The students of Teacher Y scan an electronic encyclopedia, view a film on subject, or look at
related topics at the touch of a button right there in the classroom. Which device/s does
teacher Ys class have?
A. Teaching machines
B. CD
C. Video disc
D. Videotaped lesson
10. Which is an INAPPROPRIATE way to manage off-task behavior?
A. Redirect a childs attention to task and check his progress to make sure he is continuing work
B. Make eye contact to communicate what you wish to communicate
C. Move closer to the child to make him feel your presence
D. Stop your class activity to correct a child who is no longer on task
11. To be an effective classroom manager, a teacher must be friendly but must at the same time
be_________.
.
A. confident
B. businesslike
C. analytical
D. buddy-buddy
12. Which software is needed when one wants to perform automatic calculations on numerical
data?
A. Database
B. Spreadsheet Program
C. Microsoft Word
D. Microsoft Powerpoint
13. Which of the following questions must be considered in evaluating teacher-made materials?
A. Is the material new?
B. Does the material simulate individualism?
C. Is the material expensive?
D. Is the material appropriate?
14. Kounin claims that with-it-ness is one of the characteristics of an effective classroom
manager. What is one sign of with-it-ness?
A. Giving attention to students who are having difficulty with school work
B. Seeing only a portion of the class but intensively
C. Knowing where instructional materials are kept
D. Aware of whats happening in all parts of the classroom
15. Which of these is one of the ways by which the internet enables people to browse documents
connected by hypertext links?
A. URL
B. Browser
C. Welcome page
D. World Wide Web
16. Which characteristics must be primary considered as a choice of instructional aids?
A. Stimulate and maintain students interests
B. Suited to the lesson objectives
C. Updated and relevant to Filipino setting
D. New and skillfully made
17. You can exhibit referent power on the first day of school by ________.
A. telling them the importance of good grades
B. giving your students a sense of belongingness and acceptance
C. making them feel you know what you are taking about
D. reminding your students your authority over them again and again
18. I would like to use a model to emphasize particular part. Which of these would be MOST
appropriate?
A. Realia
B. Audio recording
C. Stimulation
D. Mock up
19. What must a teacher do to ensure orderly transitions between activities?
A. Allow time for the students to socialize in between activities
B. Have the materials ready at the start of the activity
C. Assign fewer exercise to fill the allotted time
D. Wait for students who lag behind
20. The task of setting up routine activities for effective classroom management is a task that a
teacher should undertake_________. .
A. as soon as the students have adjusted to their schedules
B. on the very first day of school
C. every day at the start of the session
D. every homeroom day

ANSWER KEY:
1. C
2. B
3. C
4. C
5. C
6. D
7. B
8. D
9. A
10. D
11. A
12. B
13. D
14. D
15. B
16. B
17. C
18. D
19. B
20. B
LET Practice Test 1-Filipino

Pagsusulit Blg. 1

Panuto: Piliin ang tamang letrang sagot at ilagay ito sa inyong laki ng papel.

1. Ayaw kong palupig sa hinalang ang mga dahilang tinubos ng luha at dugo ay mga anino
lamang ng mga dantaon ay tumatawag at walang nakaririnig. Ang sanaysay na ito ay
nagpapahiwatig ng _________________.

A. kabiguan B. may pag-asa C. may kasiyahan D. may paglupig

2. Ito ang katawagan sa awiting bayan sa mga kasalan ng katutubo.

A. Ihiman B. oyayi C. diona D. soliranin

3. Mula sa Kung anong bukambibig siyang laman ng dibdib. Ito ay isang uri ng
________________.

A. Tugmaan B. bugtong C. tula D. salawikain

4. Siya ang sumulat ng Hulyo 4, 1954 A.D. na nagkamit ng unang gantimpala sa Palanca
noong 1953-1954.

A. Rogelio Sicat B. Dionisio Salazar C. Edgardo Reyes D. Amado V. Hernandez

5. Ang pinakamahaba at pinakamatandang epiko ng Panay na may 18 salaysay at sinakop ang


tatlong henerasyon.

A. Alim B. Maragtas C. Haraya D. Hinilawod

6. Alin ang kasintunog ng salitang may salungguhit? Pag-ibig ang kailangan ng daigdig.

A. Pagsuyo B. Mundo C. Tahanan D. Paligid

7. Ano ang kahulugan ng salitang may salungguhit? May mga taong bukas ang palad sa mga
mahihirap.

A. Matulungin B. Mayaman C. Matapat D. Maawain

8. Ano ang kahulugan nito? Puwit man ng basot suot ng mayaman, ang tingin ng taoy
brilyanteng makinang.
A. Mapang-api
B. Pagtingala sa mayaman
C. Mapagbalatkayo
D. Tulong sa kabuhayan
9. Alin sa mga pangungusap ang higit na epektibong pagpapahayag?

A. Nang nailikas ang mga tao ay bumaha sa aming bayan.

B. Nang bumaha sa aming bayan ay nailikas na ang mga tao.

C. Pagkalikas sa mga tao ay bumaha sa aming bayan.

D. Nang lumikas ang mga tao ay bumaha sa aming bayan

10.Kuha ito sa Ang Bayan Koy Ito, Ito ang Bayan Ko ni Jose Villa Panganiban:

Sa iisang lahi at iisang wikay magkasamang lahat

Mutya ng Silangan ay tunay ngang mutyang

Marangal ang sukat.

Alin ang ipinahihiwatig nito?

A. Matinding poot C. Dakilang damdamin

B. Mababang uri ng kaisipan D. Marahas na damdamin

11. Alin ang kahulugan ng AMPALAYANG-AMPALAYA?

A. Takot B. Kuripot C. Makulit D. Lokohin

12. Alin kaisipan ang inilalarawan ng bugtong? Akoy nagpunla ng sangkabang mais

Pagka- umagay biglang napaalis.

A. Buwan
B. Bituin
C. Bulalakaw
D. Bawang

13. Ang sumusunod ay isang kantahing bayan.

Ayokong, ayokong mag-alaga ng manok.

Pag akoy umalis pag akoy umalis,

Iba ang hahaplos

Sa aking pagdating, sa aking pagdating

Balahiboy gusot, Ito ang simula, ito ang simula

Ng sama ng loob.

Ano ang isinasaad ng awitin?

A. Ang hindi niya pagkagusto sa babaeng nagtataksil sa asawa.


B. Di niya ibig na mag-alaga ng manok na iba naman ang kakain.
C. Umiiwas siyang sumama ang loob.
D. Mahirap mag-alaga ng manok pangsabong
14. Alin Kaisipan ang inilalarawan ng bugtong na ito? Bahay ni Kiko, walang bintana, walang
pinto.

A. Langka
B. Saranggola
C. Itlog
D. Kabaong

15. Alin sa mga pangungusap ang may tamang gamit ng pantukoy na pananda?

A. Mag-aaral ang UST ang nagwagi sa timpalak.


B. Mag-aara ang UST ang nagwagi ng timpalak
C. Mag-aaral ng UST ang nagwagi sa timpalak
D. Mag-aaral ng UST ang nagwagi ng timpalak
16. Alin ang kasintunog ng salitang may salungguhit? Tinamaan ng palaso ni Kupido ang
kanyang puso.
A. Mabigo
B. Palalo
C. Katoto
D. Magaso

17. Kuha ito sa Pagsisi ng Isang Bilanggo ni Cirio H. Panganiban:

Mga magulang kong labis magpalayaw Tingnan ninyo akong labis na minahal

Kaluluwa ninyo ngayon ay nasaan? At ngayoy lugami sa hirap ng buhay.


Alin di-berbal na komunikasyon ang maaaring gamitin para makapagpapatingkad sa diwa nito?
A.Kumpas na pasubaybay C. Kumpas na paturo

B.Kumpas na pahawi D. Kumpas na pauyan na may kibitz ng balikat

18.Ano ang kahulugan ng talatang ito? Batong tuntungan mo sa pagkadakila, batong tuntungan
nang sa pamamayapa.

A. Talino
B. Kalan
C. Balatkayo
D. Monumento

19.Ang kantahing bayan na inaawit ng mga ilokano at Igorot sa mga naulila sa piling ng
bangkay ng yumaong asawa.

A. Dandansoy
B. Pamulinawen
C. Dung-aw
D. Dalit

20. Ang panahong ito ay itinuring na gintong panahon sa panitikan.

A. Amerikano
B. Aktibismo
C. Kastila
D. Hapon
Answer Key:
1. B
2. A
3. D
4. B
5. D
6. D
7. A
8. B
9. B
10. C
11. A
12. B
13. A
14. C
15. C
16. D
17. B
18. D
19. C
20. D
Practice Test in Biological Science Majorship

1. Which is the process of removing from the cells and bloodstream substances that are of no
further use to our body?
A. Respiration C. Digestion
B. Circulation D. Excretion

2. Which is an accessory organ of digestion whose function is to store and concentrate bile whic
h is secreted by the liver?
A. Spleen C. Gall bladder
B. Urinary bladder D. Pancreas

3. Identify the genetic engineering process that join cells of similar characteristics that are ofte
n closely related.
A. DNA typing C. Cloned DNA
B. Protoplast fusion D. Hybridoma

4. What is the most significant cause of dwindling biodiversity?


A. Depletion of ozone layer
B. Large plant consumption
C. Global warming
D. Destruction of habitat

5. Because she has a wide forehead, her grandma says she will be a bright student. What coul
d be inferred from his belief?
A. The cerebrum coordinates the action of all parts.
B. The frontal lobe of the brain is the cerebrum.
C. The front part is inactive when we think.
D. The thalamus is located in the sides.

6. The phenotypic ratio in the offspring resulting from a cross Tt x Tt is


A. 1:3 C. 1:2:1
B. 4:0 D. 3:1

7. Acid rain is brought about by burning tremendous amount of fossil fuel. Which are some har
mful gases that are released from burning fossil fuels?
A. Hydrogen and oxygen C. Carbon dioxide and helium
B. Sulfur dioxide and nitrogen D. Ozone and water vapor

8. Which part of the brain is responsible for thinking and reasoning?


A. Medulla C. Thalamus
B. Cerebellum D. Cerebrum

9. Name the gaseous hormone which is commonly used in inducing ripening of fruits.
A. Enzyme C. Traces
B. Methylene D. Ethylene

10. Protein is one of the most essential food nutrients needed before and after birth, otherwise i
rreversible mental retardation may occur. Which are some good sources of protein?
A. Corn, potato, bread
B. Noodles, orange, apple
C. Beans, eggs, milk
D. Garlic, pepper, eggplant

11. Which is an abiotic component that affects life in an ecosystem?


A. Animals C. Temperature
B. Plants D. Humans

12. Which is the following animals exhibits incomplete metamorphosis?


A. Butterfly C. Grasshopper
B. Moth D. Skipper

13. What processes are responsible for the occurrence of the hydrologic cycle?
A. Evaporation and sublimation
B. Transpiration and evaporation
C. Condensation and transpiration
D. Evaporation and condensation

14. Today, researchers have discovered chemical indicators in the genetic material which can tr
ace those who are predisposed to heart attack. What is this technique?
A. Irradiation C. Gene-splicing technology
B. In-breeding technique D. Linkage

15. Which describes the type of cells that all animals are made of?
A. Endoplasmic C. Compound
B. Eukaryotic D. Prokaryotic

16. The fleshy part of the cashew which we eat is its_____________.


A. Ovule C. Penduncle
B. Ovary D. Receptacle

17. Lipase aids in the digestion of______________.


A. Enzymes C. Salts
B. Proteins D. Fats

18. Cells of us unicellular organisms, like bacteria and bluegreen algae are simple in structure. T
hey are classified as_________________.
A. Eukaryotic C. Procaryotic
B. Nucleod D. Caryotic

19. Insulin which enables the liver to store sugar secreted by the______________.
A. Pituitary gland C. Islets of langerhans
B. Adrenal medulla D. Adrenal cortex

20. Which is the procedure of calculating the age of an object that contains radioactive isotopes
?
A. Rock decay C. Fossil examination
B. Radiometric dating D. Fossil indexing

21. As the test of the seed breaks, which structures come out first and grow into the first true le
aves and roots?
A. Two seed-leaves C. Plumule and radicle
B. Micropyle and epicotyl D. Testa and tegmen
22. What type of muscle occurs in the walls of hollow structure like ducts, blood vessels and int
estines whose contraction is involuntary?
A. Straited C. Smooth
B. Skeletal D. Cardiac

23. Where does exchange of materials between the blood and the cells take place?
A. Arteries C. Platelets
B. Veins D. Capillaries

24. Butterflies and mosquitoes have mouthparts in the form of a long, narrow, coiled tube calle
d__________.
A. Mandibles C. Antennae
B. Proboscis D. Filament

25. Which is the ring of muscle that serves as the opening between the stomach and the small i
ntestines?
A. Septum C. Anal
B. Sphincter D. Rectal

26. Which are the tiny organs located each segment that are used for locomotion by the earthw
orms?
A. Flagella C. Pseudopodia
B. Cilia D. Setae

27. Which lung disorder is present when the alveolar walls breakdown and gas exchange is dist
urbed?
A. Macrophage destruction C. Bronchial irritation
B. Emphysema D. Pulmonary crisis

28. Why is it not advisable to drink sea water?


A. It can result in swelling.
B. The cells will burst instantly.
C. The cells will melt.
D. It can cause dehydration of the cells.

29. It is a process of accelerated biodegradation of moist kitchen waste into a humus-


like product. What is this process?
A. Shredding C. Composting
B. Recycling D. Reuse

30. In a carbon-oxygen cycle where does the oxygen come from?


A. Animals C. Plants
B. Water D. Soil

Answer Key
1. D
2. C
3. B
4. D
5. B
6. A
7. B
8. D
9. D
10. C
11. C
12. D
13. D
14. C
15. B
16. D
17. B
18. C
19. B
20. C
21. C
22. D
23. B
24. B
25. D
26. D
27. D
28. C
29. C
30. C
Practice Test 3 in Professional Education- Situational

Situation 1- In a faculty meeting, the principal


told his teacher: We need to improve our school performance in the National Achievement Test
. What should we do?

The teacher gave varied answers as follows:


1. Lets give incentives and rewards to students who get a rating of 85%
2. Lets teach them to accept complete responsibility for their performance
3. Lets make the school environment conducive for learning
4. Lets make use of the experimental methods of teaching

1. Which response/s come/s from a behaviorist?


A. #2 and #4
B. #1 and #2
C. #3 and #4
D. #1 and #3
2. On which educational philosophy is response #1 anchored?
A. Existentialism
B. Essentialism
C. Progressivism
D. Bahaviorism
3. If you learned toward a progressivist philosophy, with which response would you agree?
A. #2
B. #3
C. #4
D. #1

Situation 2- One principle in the utilization of technology of the classroom is appropriateness


of material or activity.

4. Teacher C wants his students to master the concept of social justice. Which series of activities wi
ll be most effective?
A. Pretest-teaching-posttest
B. Pretest-teaching-posttest-re-teaching for unlearned concepts-posttest
C. Review-pretest-teaching-posttest
D. Teaching-posttest
5. Teacher A likes to show how the launching of spaceships takes place. Which of the
following materials available is most fit?
A. Model
B. Mock-up
C. Replica
D. Realia
6. Teacher B likes to concretize the abstract concepts of an atom. She came up with a
concrete presentation of the atom by using wire and plastic balls.
7. How would you classify Teacher Bs visual aids?
A. Chart
B. Replica
C. Model
D. Realia

Situation 3-
After reading and paraphrasing Robert frosts Stopping by the Wood on a snowy Evening. Mr.
Sales asked the class to share any insight derived from the poem.

8.
The class was asked to share their insights about the poem. The ability to come up with
an insight stems from the ability to .
A. analyze the parts of a whole
B. evaluate the worthiness of a thing
C. relate and organize things and ideas
D. comprehend the subject that is being studied
9. To ask the class any insight derived from the poem is based on the theory of .
A. realism
B. behaviorism
C. conditioning
D. constructivism
10. On which assumption about the learner is Mr. Marquezs act of asking the class to share their in
sight based?
A. Learners are like empty receptacles waiting to be filled up
B. Learners are meant to interact with one another
C. Learners have multiple intelligence and varied learning styles
D. Learners are producers of knowledge not only passive recipients of information

Situation 4- Principal E wants her teachers to apply constructivism in teaching

11. On which assumption/s is the principals action anchored?


I. Students learn by personally constructing meaning of what is taught.
II. Students are construct and reconstruct meaning based on experiences
III. Students derive meaning from the meaning that the teacher gives
A. II only
B. I and II
C. I, II, and III
D. I only
12. Which materials will her teachers LEAST prefers?
A. Controversial issues
B. Open-ended topics
C. Unquestionable laws
D. Problem or cases
13. Which concept/s of the learner will Principal E NOT accept?
I. Empty vesse!
II. Tabula rasa
III. Candle to be lighted
A. III only
B. I only
C. II only
D. I and II

Situation 5- Study the matching type of test then answer the 3 questions that follow:

Column A Column B
1. equilateral triangle A. With 3 equal sides
2. right triangle B. With 5 equal sides
3. octagon C. Has 90- degree angle
4. pentagon D. Means many
5. heptagon E. with 7 sides
6. poly F. with 8 sides

14. How can you make the items homogeneous?


A. Increase the number of items in Column B
B. All items should be on polygons
C. Remove the word triangle in items #1 and #2 in column A
D. The word gon must be included in column B
15. What is the main defect of this matching test?
A. the matching type is an imperfect type
B. the items are NOT homogeneous
C. the items quite easy
D. an obvious pattern is followed in the answering
16. Which should be done to improve the matching type of test?
A. Capitalize the items in Column A
B. Items in Column A and B should be exchanged
C. Drop #6 item in Column A
D. The item in Column A should be increased

Situation 6- Below the template for Scoring Rubric.


5-Demonstrate complete understanding of the problem.
All requirements of task are included in response
4-Demonstrate considerable understanding of the problem.
All requirements of task are included
3- Demonstrate partial understanding of the problem.
Most requirements of task are included
2- Demonstrate little understanding of the problem.
Many requirements of task are missing
1- Demonstrate no understanding of the problem
0-No response/task not attempted

17. Which of these is/are essential in constructing a scoring rubric?


I. Description of criteria to serve as standard
II. Clear descriptions of performance at each level
III. Levels of achievement (mastery)
IV. Rating scheme
A. I, II, III
B. I, II
C. I, II, III, IV
D. I only
18. Which statement is TRUE of the rubric?
A. It is developmental
B. It is analytical
C. It is both holistic and developmental
D. It is holistic
19. Which is TRUE of the scoring rubric?
I. It describes criteria of levels of achievement
II. It has a rating scheme
III. It limit itself to 4 levels of achievement
A. I and II
B. I and III
C. II and III
D. I, II and III

Situation 7- Study the table on item analysis for non-attractiveness and


non-plausibility of distracters based on the results of a tryout test in Science.
The letter marked with a asterisk is the correct answer.

Item No. 1 A B C D E
Upper 27% 10 4 1 1 0
Lower 27% 6 5 2 2 0

20. The table shows that the test item analyzed .


A. has a positive discrimination index
B. has a negative discrimination index
C. is extremely easy
D. is extremely difficult
21. Based on the table, which is the most effective distracter?
A. Option D
B. Option A
C. Option C
D. Option B
22. Based on the table, which group got more correct answer?
A. Upper group
B. It cannot be determined
C. Lower group
D. Data are not sufficient to give an answer

Answer Key:
1.D
2.D
3.C
4.B
5.B
7.C
8.D
9.D
10.D
11. B
12. C
13. A
14. A
15. B
16. B
17. C
18. C
19. D
20. B
21. B
22. A
Practice Test 4 in Professional Education

1. On which constitutional provision is the full or partial integration of capable deaf and blind in
the classroom based?
The provision ______.
A. academic freedom for students
B. providing vocational training to adult citizens and out-of-school youths
C. creating scholarship for poor and deserving students
D. protecting and promoting the right of all citizens to quality education

2. Teacher A put together the output of her colleagues in one workshop and published it with h
er name as author.
Which is unprofessional about Teacher As behavior?
A. Not giving due credit to others for their work
B. Failing to correct what appears to be unprofessional conduct
C. Giving due credit to others for their work.
D. Holding inviolate all confidential information concerning associates

3. I want every pupil to cope with my lesson. Which technique can help me achieve it?
A. Peer instruction
B. Film showing
C. Mnemonics
D. Choral reading

4. Who did an analysis of a Greek tragedy which served as a model for literary criticism?
A. Pythagoras
B. Aristotle
C. Plato
D. Socrates

5. Teacher B is researching on family income distribution which is quite symmetrical. Which me


asure/s of central tendency will be most informative and appropriate?
A. Mode
B. Mean
C. Median
D. Mean and Median

6. In what way can teachers uphold the highest possible standards of the teaching profession?
A. By working for promoting in the system
B. By continuously improving themselves personally and professionally
C. By pointing out the advantages of joining the teaching profession.
D. By good grooming to change peoples poor perception of teachers

7. Which way to happiness did Aristotle and Maslow share in common?


A. Satisfaction of all our needs
B. Fulfillment of our obligations and duties
C. Spontaneity and simplicity in our way of life
D. Realization of our potentials to the fullest
8. I like to develop the synthesizing skills of my students. Which one should I do?
A. Ask my students to formulate a generalization from the data shown in the graphs.
B. Direct my students to point out which part of the graph is right and which part is wrong.
C. Ask my students to answer the questions beginning with What if
D. Tell my students to state data presented in the graph.

9. I want to my students a conceptual preview of what I am to teach. Which one will I use?
A. Advance organizer
B. Problem tree
C. Mnemonic device
D. Venn diagram

10. Which type of questions are generally considered low-level?


A. Convergent
B. Evaluative
C. Divergent
D. Open-ended

11. Study this group of tests which was administered with the following results, then answer th
e question:
SUBJECT MEAN SD RONNELS SCORE
Math 40 3 58
Physics 38 4 45
English 75 5 90

In which subject(s) were the scores most homogenous?


A. English
B. Physics
C. Math and English
D. Math

12. In which way does heredity affect the development of the learner?
A. By placing limits beyond which the learner cannot develop
B. By making acquired traits hereditary
C. By compensating for what environment fails to develop
D. By providing equal potential to all

13. A teacher should not be a slave of his lesson plan. This means that _____________.
A. the teacher must be ready to depart from her lesson plan if she remembers something more
interesting
than what she planned
B. a lesson must be followed by a teacher no matter what
C. the teacher is the best lesson plan designer
D. the teacher must be willing to depart from her lesson plan if students are more interested in
something other than in what she planned

14. Which is a typical characteristic of the adolescent?


A. Reasonable and secure
B. Emotional unstable
C. Slow but steady physical growth
D. Passive and obedient

15. Which one is meant to measure students awareness of values?


A. Anecdotal record
B. Projective technique
C. Moral dilemma
D. Likert scales
16. The first American teachers in the Philippines were ______________.
A. elementary graduates
B. soldiers
C. missionaries
D. graduates of the normal school

17. An athlete student is bored and restless. Teacher Justin incorporates athletics into the discu
ssion to get his attention.
Which disciplinary technique does Teacher Justin use?
A. Signal interference
B. Direct appeal
C. Interest boosting
D. Hurdle lessons

18. The difficulty index of a test item is 1. What does this imply? The test item must be ______
___.
A. moderate in difficulty
B. very difficult because only 1 got the item correctly
C. very easy because everybody got the test item correctly
D. neither difficult nor easy

19. For global competitiveness, a school must embark on proactive change.


Which one is a characteristic of a proactive change?
A. Imitative of others
B. Problem-driven
C. Late in the game
D. Radical and inventive

20. Which test item is in the highest level of Blooms Taxonomy of objectives?
A. Explain how a tree functions in relation to the ecosystem
B. Explain how trees receive nutrients.
C. Rate 3 diff. methods of controlling tree growth
D. List the parts of a tree.

Answer key:
1.D
2.A
3.A
4.B
5.D
6.B
7.D
8.A
9.A
10.D
11.B
12.A
13.A
14.B
15.C
16.C
17.C
18.C
19.D
20.A
PRACTICE TEST IN LET ENGLISH-WORD ANALOGY

1. GNASH is related to TEETH as LISTEN is related to ____________.


a. hear
b. ears
c. resolve
d. dissuade
2. BREAD is related to BUTTER as POTATOES is related to ____________.
a. gravy
b. steak
c. margarine
d. lamb
3. PORCINE is related to PIG as BOVINE is related to ____________.
a. boy
b. cow
c. sheep
d. iodine
4. TAILOR is related to NEEDLE as MECHANIC is related to ____________.
a. engineer
b. screwdriver
c. tool chest
d. brush
5. DISTANCE is related to MILE as LIQUID is related to ____________.
a. milk
b. quart
c. water
d. meter
6. AUTOMOBILE is related to HIGHWAY as LOCOMOTIVE is related to ____________.
a. station
b. train
c. track
d. engine
7. SUGGEST is related to REQUIRED as REQUEST is related to ____________.
a. ask
b. demand
c. suspect
d. allow
8. PROBLEM is related to SOLUTION as POISON is related to ____________.
a. hemlock
b. nitrate
c. arsenic
d. antidote
9. SAW is related to CUT as YARDSTICK is related to ____________.
a. foot
b. inch
c. measure
d. tool
10. AMBIGUOUS is related to CLARITY as TEMPORARY is related to ____________.
a. transient
b. permanence
c. clear
d. fragile
11. BOOK is related to CHAPTER as SONG is related to ____________.
a. stanza
b. sing
c. music
d. instrument
12. DELICACY is related to GOURMET as INSECT is related to ____________.
a. bee
b. pollen
c. frog
d. hive
13. AGILE is related to NIMBLE as FAST is related to ____________.
a. swift
b. slow
c. perpetual
d. racy
14. CHEMIST is related to LABORATORY as ARTIST is related to ____________.
a. canvas
b. museum
c. easel
d. studio
15. MOVIES is related to PROJECTOR as RECORDS is related to ____________.
a. speakers
b. tape recorder
c. phonograph
d. radio
16. STOOPED is related to POSTURE as SLURRED is related to ____________.
a. diction
b. stance
c. music
d. action
17. RETRACT is related to STATEMENT as VOID is related to ____________.
a. escape
b. avoidance
c. contract
d. empty
18. FELONY is related to MISDEMEANOR as KILL is related to ____________.
a. maim
b. bury
c. murder
d. guilty
19. BEGGAR is related to POOR as FOX is related to ____________.
a. skunk
b. slow
c. large
d. sly
20. SCALPEL is related to KNIFE as NURSE is related to ____________.
a. doctor
b. assistant
c. hospital
d. operation

ANSWER KEY and EXPLANATION


1. B The first word is the action of the second word.
2. A The second is usually put on the first word.
3. B The first word means pertaining to the second word.
4. B The second word is a tool of the first word.
5. B The second word is a measure of the first word.
6. C The first word travels on the second word.
7. B The second word is an imperative (must) of the first word.
8. D The second word overcomes the first word.
9. C The first word is the tool to do the second word.
10. B The first word is the opposite of the second word.
11. A The second word is a part of the first word.
12. C The first word is eaten by the second word.
13. A The first word is a synonym of the second word.
14. D The second word is where the first word works.
15. C The second word plays the first word.
16. A The first word is a n impairment of the second word.
17. C The first word nullifies the second word.
18. A The first word is a more serious degree of the second word.
19. D The second word is a characteristic of the first word.
20. B The first word is a medical term for the second word.
Practice Test in LET Biological Science-majorship

1. Where do producers obtain their energy?

A. From other producers

B. From the consumers

C. From the decomposers

D. From the sun

2. Why do you experience not seeing things clearly for some seconds when you enter suddenly a
well-lighted room after coming from a dark room, you?

A. The pupils are not adapted to the dark

B. The eyes are adapted to the light only

C. The pupils are not dilated yet

D. Light had caused temporary blindness

3. What term is used to describe the ability to maintain a constant internal environment?

A. Metabolism

B. Growth and development

C. Homeostasis

D. Thermoregulation

4.What is the most common cause of cardiovascular disease?


A. Fatty deposits in the arteries

B. Inadequate supply of red blood cells

C. Lack of sodium in the diet

D. Increased heartbeat

5.Which is the smallest unit of life that can survive and reproduce on its own?

A. Cell

B. Organ

C. Tissues

D. Population

6.Which region of a vertebrate forebrain is considered with the neutral-endocrine control of visceral
activities?

A. Cerebellum

B. Thalamus

C. Hypothalamus

D. Pituitary

7.Which of the following is the building block carbohydrate?

A. Amino acid

B. Nucleotide

C. Fatty acid

D. Monosaccharide

8.Why it is necessary men have more red blood cells per cubic millimeter of blood than women?

A. Man does strenuous activities and need more energy

B. They have bigger physique and need more blood

C. Man waste more blood cells and need extra

D. They are more prone to anemia disorders


9.Which of the following is the building block of protein?

A. Amino acid

B. Nucleotide

C. Fatty acid

D. Monosaccharide

10.Where does protein synthesis take place?

A. Ribosomes

B. Lysosomes

C. Endoplasmic reticulum

D. Golgi bodies

11.Why are our lips redder than our palm?

A. There is increased blood flow in the lips

B. Lips are heavily keratinized

C. Lip epidermis is thinner

D. Many blood vessels are located around the lips

12.Which of the following cannot be found in prokaryotes?

A. Cytoplasm

B. Plasma membrane

C. Membrane-bound nucleus

D. DNA

13.What do we call mutation that is brought about by the earth natural radioactivity?

A. Continuous

B. Induced

C. Lysosomes

D. Chloroplast
14.Which of the following organelles produces turbo pressure against cell wall and mainly acts as
water reservoir

A. Plastids

B. Vacuole

C. Lysosomes

D. Chloroplast

15.What is the branch crobial organisms is used in the creation of genetically modified organisms?

A. Pseudomonas vulgaris

B. Bacillus thuringensis

C. Agro bacterium tumafaciens

D. Escherichia coli

16.What is the branch of zoology that is devoted to the study of fish?

A. Helmintology

B. Ichthyology

C. Chondrichthyes

D. Ornithology

17.The process involved in the production of mRNA using DNA as template is termed _______?

A. Transcription

B. Replication

C. Gene manipulation

D. Reverse transcription

18.What is the most abundant inorganic compound in the protoplasm?

A. Fluid

B. Blood

C. Plasma
D. Water

19.Which is the riches type of tropical rain forest in the Philippines?

A. Molave forest

B. Dipterocarp forest

C. Mangrove forest

D. Pine forest

20.What muscle is described as involuntary, not striated, and have a single nucleus?

A. Smooth

B. Skeletal

C. Cardiac

D. Connective

21.We feel warmer just before it rains because __________.

A. Heat is released by the evaporation of water

B. The clouds prevent heat from escaping the earth

C. There is an increased relative humidity

D. Heat is released by the condensation of water vapor

22.Which of the following is not considered as a reproductive cell?

A. Gamete

B. Somatic cell

C. Egg cell

D. Sperm cell

23.The cell wall of plant is made of _______.

A. Lipids

B. Cellulose

C. Protein
D. Cell membrane

24.Which of the following is the correct sequence of the level of organization from lowest to
highest?

A. Cells-system-organs-tissues

B. System-organs-tissues-cells

C. Cells-tissues-organs-system

D. Tissues-systems-cells-organs

25.At metaphase of the meiotic division, the chromosomes are attached to the spindle fibers as
________.

A. Double chromatids

B. Non-homologous pairs

C. Single chromatid strands

D. A tetrad of four chromatids

26.Which of the following is an example of behavioral adaption?

A. Thick green stems of the cactus plant

B. Thick fur of the bear

C. Shedding leaves during summer

D. Presence of cuticle on the upper surface of the leaves

27.The bipolar nature of the cell membrane is due to _______.

A. Presence of carries

B. Presence of

C. Phospholipids bilayer

D. Integral proteins

28.Which of the following is made up of diploid number of cells?

A. Zygote
B. Sperm cell

C. Egg cell

D. Both B and C

29.Which plant has underground stem modified for reproduction?

A. Ginger

B. Camote

C. Tubers

D. Strawberry

30.Which of the following structures serve as a passageway of food and air?

A. Epiglottis

B. Trachea

C. Pharynx

D. Larynx

31.Which kind of relationship is exhibited by algae and fungi in lichen?

A. Commensalism

B. Parasitism

C. Symbiosis

D. Competition

32.What do you call the finger-like folds on the inner linings of the small intestine?

A. Villi

B. Appendix

C. Rugae

D. Cecum

33.Aside from ADP, what else is the end production of the dark-reaction phase of ohotosynthesis?

A. Carbon dioxide
B. Surag

C. Carbon

D. Starch

34.What is that flap-like structure that prevents the food from going the wrong way during
swallowing?

A. Epiglottis

B. Larynx

C. Esophagus

D. Pharynx

35.Which pigment is dominant in red algae?

A. Phycocyanin

B. Fucoxanthin

C. Chlorophyll

D. Pharynx

36.What is the membrane that surrounds the lungs?

A. Pleura

B. Meninges

C. Peritoneum

D. Pericardium

37.What is that 3-carbon sugar formed during the dark reaction phase of the photosynthesis?

A. ADP

B. NADPH

C. ATP

D. PGAL

38.What is the product of carbohydrate digestion?


A. Fatty acid

B. Glucose

C. Amino acid

D. Nucleic acid

39.What process is responsible for the upward movement of the water in very tall trees?

A. Osmosis

B. Capillary action

C. Turgor pressure

D. Transpiration

40.What is the enzyme in the mouth that breaks starch to maltose?

A. Peptidase

B. Ptyalin

C. Pepsin

D. Maltase

41.What do you call the types of symmetry where the body parts are paired on either side of the
body?

A. Dorsal

B. Bilateral

C. Radial

D. Ventral

42.Which is considered as the respiratory center of the brain?

A. Cerebellum

B. Medulla oblongata

C. Cerebrum

D. Thalamus
43.Which stage of incomplete metamorphism is undergone by grasshopper?

A. Nymph, pupa, adult

B. Egg, larva, pupa, adult

C. Egg, nymph, adult

D. Egg, pupa, adult

44.Which of the following is NOT part of the thoracic cage?

A. Clavicle

B. Ribs

C. Costal cartilages

D. Sternum

45.Where does the toxin substance secreted by bees and ants through their sting
originate?

A. Silk glands

B. Intestines

C. Rectal glands

D. Salivary glands

46.What is the longest bone in the body?

A. Humerus

B. Ferum

C. Tibia-fibula

D. Clavicle

47.Which cause our bones to turn brittle and easily break?

A. Turning into muscle tissues

B. Increase in flexibility

C. Turbidity decrease

D. Removal of collagen
48.How many bones does an adult human body has?

A. 201

B. 200

C. 217

D. 206

49.What do you call automatic responses to an external stimulus?

A. Reflex

B. Instinct

C. Impulse

D. All of these

50.What structure provides flexible support and protection to the spinal


cord?

A. Skull

B. Ribs

C. Vertebral column

D. Sternum

51. Which hormone is insufficient in


cretinism?

A. Glucagon

B. Adrenalin

C. Insulin

D. Thyroxine

52.Which of the following is both endocrine and an exocrine


gland?

A. Gastric glands

B. Thyroid glands
C. Pituitary glands

D. Pancreas

53.What organ is controlled by the automatic nervous


system?

A. Skeletal

B. Muscles

C. Heart

D. Both A and B

54.What do you call food in a semi-liquid form, partially digested and mixed with hydrochloric
acid?

A. Gastric secretion

B. Gastric fluid

C. Mucosa

D. Chime

55.Which is considered the master gland due to its influence on the activity of all the other
glands?

A. Thyroid

B. Adrenal

C. Pituitary

D. Parathyroid

56.Which part of the brain is responsible for intelligence, memory and learned
behavior?

A. Cerebellum

B. Cerebrum

C. Medulla

D. Thalamus
57. Which of these materials is not a major component of the plasma
membrane?

A. Phospholipids

B. Glycoprotein

C. Proteins

D. DNA

58.Which form of RNA delivers information from DNA to be used in making


protein?

A. Messenger RNA

B. Ribosomal RNA

C. Transfer RNA

D. All of the above

59.Which of the following is not a type of


cell?

A. Bacterium

B. Amoeba

C. Sperm

D. Virus

60.Which property of water is probably MOST important for the functioning of organisms at the
molecular level?

A. Cohesion and high surface tension

B. High specific heat

C. High heat of vaporization

D. Versatility as a solvent

61.Which of the following statements is true of


diffusion?

A. It requires an expenditure of energy by the cell

B. It is a passive process
C. It occurs when molecules move from a region of lower concentration to one of higher
concentration

D. It requires a membrane

62.Which of the following is FALSE in comparing prophase I of meiosis and prophase of


mitosis?

A. The chromosomes condense in both

B. Tetrads form in both

C. The nuclear envelope disassembles in both

D. A spindle forms in both

63.Which are small, irregularly-shaped cells without nuclei but are rich in
ATP?

A. Leukocytes

B. Thrombocytes

C. Lymphocytes

D. Erythrocytes

64.What branch of Biology is involved when one is studying the characteristic structure and
functions of all kinds of cells?

A. Cytology

B. Morphology

C. Physiology

D. Histology

65.Which part of seed appears as a scar which marks the entry of the pollen tube during
fertilization?

A. Seed leaf

B. Cotyledon

C. Testa

D. Hilum
66.Which type of tissue is responsible for the secondary growth in stems and roots of vascular
plants?

A. Meristematic cells

B. Cambium

C. Palisade layer

D. Epidermis

67.Which biome grows tropical grassland with scattered individual trees and larger
herbivores?

A. Tundra

B. Rain forest

C. Grassland

D. Savanna

68. What do you call that process in birds wherein they shed off features at least once a
year?

A. Skinning

B. Molting

C. Furrowing

D. Migrating

69.What chemical controls metamorphosis among


insects?

A. Enzymes

B. Estradol

C. Pheromones

D. Ethylene

70.Which of the following makes up the plant cell


wall?

A. Nucleic acid

B. Cellulose
C. Protein

D. Lipids

71.Which of the following is not true of


fungi?

A. Some are pathogenic

B. Some are photosynthetic

C. Some are edible

D. Form symbiotic relation with algae

72.Which of the following is not true of


plants?

A. Some are pathogenic

B. Some are photosynthetic

C. Some are edible

D. Form symbiotic relation with algae

73.Which of the following is considered multi-


cellular?

A. Embryo

B. Fertilized egg

C. Egg cell

D. Sperm cell

74.What hormone promotes the growth of the uterine lining for the implantation of the
embryo?

A. Lactogenic hormone

B. Progesterone

C. Testosterone

D. Oxytocen
75.Which of the following pair of organisms are closely
related?

A. Spider and mosquito

B. Frog and crocodile

C. Man and monkey

D. Ants and aphids

76.What does it mean if the blood pressure of human is


110/70?

A. The systolic pressure is 70

B. The diastolic pressure is 70

C. The pulse rate is 110 beats per minute

D. Both B and C

77.Which of the following happens when man


exhale?

A. The residual volume of the lungs decreases

B. The diaphragm contracts

C. The volume of the thoracic cavity decreases

D. The lung contracts

78.Which of the following regulate


breathing?

A. RBC concentration

B. Number of haemoglobin

C. Hormone level in blood

D. CO2 and O2 concentration and pH level sensors

79.Which of the following has the lowest velocity of blood


flow

A. Veins

B. Arteries
C. Capillaries

D. Arterioles

80. How many carbon atoms can each pyruvic acid supply into the Kirbs
cycle?

A. 8

B. 6

C. 4

D. 2

81.How many oxygen is required each time to a molecule of glucose is completely oxidized through
aerobic respiration?

A. 3

B. 6

C. 12

D. 24

82.Which of the following will not be accepted by glycolysis for


catabolism?

A. Starch

B. Glycerol

C. Fatty acid

D. Sucrose

83.Which of the following characteristic is common between echinoderms and


cnidarians?

A. They are both radically symmetrical

B. They have segmented bodies

C. Both have stinging cells

D. They have three embryonic tissue layers


84.Which of the following is true of Phylum
Chordata?

I.Presence of notochord that provides skeletal support

II. Pharyngeal slits that have become modified for gas exchange

III. Dorsal hallow nerve cord that develops into the central nervous system

A. II and III

B. I and II

C. I and III

D. I, II and III

85.Which of the following cannot be associated with


fungi?

A. Absorptive nutrition

B. Decomposers

C. Autotrophs

D. Spore production

86.Which of the following does not involve


mitosis?

A. Development of embryo

B. Growth

C. Production of gametes

D. Repairing of damaged tissues

87.Which of the followings is true of


mammals?

I. They have four chambered heart

II. They produce milk for the young

III. They are warm blooded

A. I and II

B. II and III

C. I and III
D. I, II and III

88.What term describes an egg develop into a new organism without itself being fertilized be a
sperm?

A. Regeneration

B. Conjugation

C. Parthenogenesis

D. Fusion

89.Which of the following is responsible for formation of blood


cell?

A. Cartilage

B. Bone narrow

C. Joints

D. Tendons

90.What is most likely happen when a species cannot adapt to the changes in the
environment?

A. It will be transform to another form

B. It will become extinct

C. It will be isolated

D. It will grow old

91.What term is used to describe the released of matured egg cell from the
ovary?

A. Fertilization

B. Copulation

C. Ovulation

D. Germination

92. Which of the following blood cells is responsible for blood


clotting?

A. Erythrocytes
B. Leukocytes

C. Thrombocytes

D. Neutrophils

93.Where does ectopic pregnancy


occur?

A. Uterus

B. Ovary

C. Cervix

D. Fallopian tube

94.Why is cell membrane semi permeability


important?

A. It is important in determining the size of the cell

B. It controls the kind of substances that enters and leaves the cell

C. It controls the pressure and energy gradient

D. It allows the molecules to diffuse freely in and out of the cell

95.What theory of evolution accounts for snakes disappearance of legs and development of
giraffes long neck?

A. Theory of use and disuse

B. Theory of natural selection

C. Theory of chromosomal change

D. Theory of action and interaction

96.Which of the following is an exocrine


gland?

A. Plastids and pigments

B. Asters and centrioles

C. Chloroplasts and vacuoles

D. Cell wall and cell membrane


97.Which of the following is an exocrine
gland?

A. Thyroid gland

B. Salivary gland

C. Pituitary gland

D. Adrenal gland

98.Which of the following is not an accessory organ of


digestion?

A. Liver

B. Pancreas

C. Salivary gland

D. Adrenal gland

99.What hormone stimulates the development of male secondary sex


characteristics?

A. Estrogen

B. Prolactin

C. Testosterone

D. Progesteron

100.Which of the following parts performs both as reproductive and urinary functions in
males?

A. Ureter

B. Testes

C. Urethra

D. Scrotum

ANSWER KEY

1. D

2. C
3. C

4. A

5. A

6. C

7. D

8. A

9. A

10. A

11. C

12. C

13. C

14. B

15. D

16. B

17. A

18. D

19. B

20. A

21. D

22. B

23. B

24. C

25. A

26. C

27. C

28. A

29. C

30. C

31. C
32. A

33. B

34. A

35. D

36. D

37. D

38. B

39. B

40. B

41. B

42. B

43. C

44. A

45. C

46. B

47. D

48. D

49. A

50. C

51. D

52. D

53. C

54. D

55. C

56. B

57. D

58. A

59. D

60. D

61. B
62. B

63. D

64. A

65. D

66. B

67. D

68. B

69. A

70. B

71. B

72. D

73. A

74. B

75. C

76. B

77. C

78. D

79. A

80. D

81. B

82. C

83. A

84. D

85. C

86. C

87. D

88. C

89. B

90. B
91. C

92. C

93. D

94. B

95. A

96. B

97. B

98. D

99. C

100. C

Practice Test 5 in Professional Education

1. School curriculum reflects the worlds economic and political integration and industrialization.
what does these point in curriculum development.

A. The trend towards the classical approach to curriculum development


B. The trend toward the globalization and localization
C. The trend toward participatory curriculum development
D. The shift in the paradigm of curriculum development from a process-oriented to a product-
oriented one

2. You choose cooperative learning as a teaching approach. What thought is impressed on your
students?
A. Interaction is a must, but not necessarily face to-face interaction
B. Students success depends on the success of the group
C. Students individuality evaluate how effectively their group worked.
D. The accountability for learning is on the group not on the individual

3. What principle is violated by overusing the chalkboard, as though it is the only education
technology available?
A. Isolated use
B. Flexibility
C. Variety
D. Uniformity

4. When more senses are stimulated, teaching and learning become more effective. What is an
application of this principle?
A. Appeal to students sense of imagination
B. Use multisensory aids
C. Make your students touch the instructional material
D. Use audiovisual aids because the eyes and the ears are the most important senses in learning.
5. Which is a classroom application of the theory of operant conditioning?
A. Help student see the connectedness of facts, concepts, and principles
B. Create a classroom atmosphere that elicits relaxation
C. Reinforce a good behavior to increase the likelihood that the learner will repeat the response
D. Make students learn by operating manipulatives

6. Here is the test item.


From the data presented in the table, form generalizations that are supported by
the data.
Under what type of question does this item fall?
A. Convergent
B. Evaluative
C. Application
D. Divergent

7. I want to teach concepts, patterns and abstractions. Which method will be MOST appropriate?
A. Discovery
B. Indirect instruction
C. Direct instruction
D. Problem solving

8. Teacher A teaches English as a Second Language. She uses vocabulary cards, fill-in-the-blanks
sentences, dialogues, dictation and writing exercises in teaching a lesson about grocery
shopping. Based on this information, which of the following is a valid conclusion?
A. The teacher wants to make her teaching easier by having less talk
B. The teacher emphasizing reading and writing skills
C. The teacher is teaching in a variety of ways because not all students learn in the same manner
D. The teacher is applying Blooms hierarchy of cognitive learning

9. Teacher A an experienced teacher, does daily review of past lessons in order to _____
.
A. introduce a new lesson
B. reflect on how he presented the previous lessons
C. provide his pupils with a sense of continuity
D. determine who among his pupils are studying

10. To teach the democratic process to the pupils, Biag Elementary School decided that the
election of class officers shall be patterned after local elections. There are qualifications set for
candidates, limited period for campaign and rules for posting campaign materials, etc. Which of
the following did the school use?
A. Symposium
B. Simulation
C. Role playing
D. Philips 66

11. Teacher S teaches a lesson in which students must recognize that is the same 0.25. They
use this relationship to determine that 0.15 and 0.20 are slightly less than . Which of the
following concept/s is/are being taught?
A. Numeration skills
B. Place value of decimals
C. Numeration skills of decimals and relationships between fractions and decimals
D. Relationship between fraction and decimals

12. To nurture students creativity, which activity should a teacher AVOID?


A. Ask hat if questions
B. Ask divergent thinking questions
C. Emphasize the need to give right answers
D. Be open to out-of-this-world ideas

13. After reading an essay. Teacher B wants to help sharpen her students ability to interpret.
Which of these activities will be most appropriate?
A. Drawing conclusions
B. Making inferences
C. Getting the main idea
D. Listing facts separately from opinion

14. Bruners theory on intellectual development moves from inactive to iconic and symbolic stages.
Applying Bruners theory. How would you teach?
A. Be interactive in approach
B. Begin with the abstract
C. Begin with the concrete
D. do direct instruction

15. In a Social studies class. Teacher I presents a morally ambiguous situation and asks student
what they would do. On whose theory is Teacher Is technique based?
A. Bandura
B. Piaget
C. Kohberg
D. Bruner

16. What does Gagnes hierarchy theory propose for effective instruction?
A. Be concerned with the socio-emotional climate in the classroom
B. Teach beginning with the concrete
C. Sequence instruction
D. Reward good behavior

17. Based on Freuds theory, which operate/s when a student strikes a classmates at the height of
anger?
A. Ego
B. Id
C. Id and Ego interact
D. Superego

18. With assessment of affective learning in mind, which does NOT belong to the group?
A. Cloze test
B. Moral dilemma
C. Reflective writing
D. Diary entry
19. If a teacher wants to measure her students ability to discriminate, which of these is an
appropriate type of test item as implied by the direction?
A. Outline the Chapter on The Cell.
B. Summarize the lesson yesterday.
C. Group the following items according to shape.
D. State a set of principle that can explain the following events.

20. Which can be said of Arielle who obtained a score of 75 out of 100 items in a Grammar
objective test?
A. She performed better than 25% of her classmates
B. She answered 75 items in the test correctly
C. Her rating is 75
D. She answered 75% of the test items correctly

Answer Key:

1. B
2. B
3. B
4. B
5. C
6. D
7. D
8. C
9. C
10. B
Practice Test in English-LET

1. The tribulations that I faced were _______ for me to overcome.


A. So much
B. Too much
C. Very much
D. So more

2. In Benjamin Fraklins Poor Richards Almanac it was said that: early to bed, _______
makes a man healthy
A. Early rising
B. Rising early
C. Early to rise
D. Waking up early

3. Which among these words has the voiceless /th/ sound?


A. This
B. Gather
C. Think
D. There

4. Which word must have the primary stress in the sentence below if we want to emphasize
possessions?
That is my boyfriend
A. That
B. Is
C. My
D. Bag

5. I would not leave you come hell or high water. Based on the given sentence, we can surmise
that:
A. The speaker wont leave the person no matter what.
B. The speaker warns the person of a forthcoming disaster.
C. The speaker wants the person to give up
D. The speaker wants the person to continue fighting.

6. Which does not belong?


A. Assumption
B. Summary
C. Synthesis
D. Generalization

7. Complete the statement: Blessed are _____ poor for _____ shall be welcomed in heaven.
A. The-you
B. The-they
C. You-the
D. They-they

8. Your brother isnt supporting you, ______?


A. Isnt she
B. Is she
C. Arent she
D. Is he

9. The Rubaiyat has a theme:


A. Always looking forward a new day
B. Never giving in to death easily
C. Grasping pleasure while you can
D. Creating your own world and beautify it

10. What is True Learning as exemplified in this quotation: Only one who burst with eagerness
do I instruct. Only one who bubbles with excitement do I enlighten.
A. Learning makes human beings a follower of instructions
B. Learning involves patience and tenacity
C. Learning has the ability to make a man
D. Learning is a coupled with commitment and passion.

ANSWER KEY:

1. B

2. C

3. C

4. C

5. A

6. A

7. B

8. D

9. C

10. D
Practice Test in General Education Mathematics-LET

1. What is the missing term in the series? 2, 5, 11, 23, 47, ____
a. 115 b. 95 c. 105 d. 125
2. A store owner makes a 25% profit by selling an item for P800.00. How much is his profit?
a. P 1,400.00 b. P 200.00 c. P 250.00 d. P 160.00
3. The simplest expressions for 25/100 is ______
a. 1 b. 4 c. d. 1/4
4. If the ratio of women to men in a meeting is 4 to 1, what percent of the persons in the
meeting are men?
a. 80% b. 20% c. 33 1/3 % d. 25%
5. Joel repacks a 60 kg sack of sugar into small packs of 750 g. How many small packs can be
made?
a. 90 b. 45 c. 55 d. 80
6. How much bigger is 550 than 375?
a. 175 b. 73 c. 45 d. 431
8. The product of 2 numbers is 36, and their ratio is 1: 4. Which of these is the smaller
number?
a. 9 b. 12 c. 3 d. 2
9. An Arithmetic book is 2 cm thick. How many copies can be placed in a meter long shelf?
a. 44 b. 40 c. 36 d. 45
10. What is the smallest positive number that is a multiple of both 12 and 14?
a. 36 b. 42 c. 84 d. 168
11. 40% of 35 is what percent of 140?
a. 28% b. 29% c. 10% d. 14%
12. An employee earning P 9,200.00 a month will receive 15% increase next month. How much
will his/her new salary be?
a. P10,500 b. P10,530 c. P10,580 d. 10,560
13. In a certain school, the ratio of boys to girls is 3 to 7. If there are 150 boys and girls in the
school, how many boys are there?
a. 45 b. 90 c. 105 d. 75
14. A park has a triangular shape. It has a base of 12m and a height of 9m. What is the area?
a. 122 sq. m b. 42 sq. m. c. 108 sq. m. d. 54 sq. m.
15. The length of a new piece of chalk is about _________.
a. 1mm b. 10mm c. 10cm d. 1 cm
16. If 50% of x is 20, what is 30% of x?
a. 30 b. 12 c. 20 d. 16
17. Which of these fractions has the greatest value?
a. 4/5 b. 9/16 c. 7/10 d. 5/8

18. Which of the following is exactly divisible by 3 and 11?


a. 357404 b. 114345 c. 991111 d. 135792
19. If the variance of distribution is 25, then the standard deviation is _________.
a. 30 b. 5 c. 625 d. 12.5
20. Which of these numbers is greater than ?
a. 0.04 b.0.09 c. 1/8 d. 1/0.04

23. A senior class of 50 girls and 70 boys sponsored a dance. If 40% of the girls and 50% of the
boys attended the dance, approximately what percent attended?
a. 44 b. 46 c. 42 d. 40
24. What is the smallest positive integer that has 6, 8, and 10 as factors?
a. 300 b. 240 c. 80 d. 120
25. Joseph has more money than Mila but less than Billy. If the amounts held by Joseph, Mila and
Billy are x, y, and z respectively, which of the following is TRUE?
a. Z<X<Y b. X<Z<Y c. Y<X<Z d. Y<Z<X
26. How many ounces of pure acid must be added to 20 ounces of a solution that is 5% acid in
strength?
a. 2 b. 1 c. 3 d. 5

28. Michael is 15 years older than this brother Rafael. However, y years ago Michael was twice as
old as Rafael. If Rafael is now x years old and x>y, find the value of x y.
a. 13 b. 14 c. 15 d. 16
29. What percent is of ?
a. 65% b. 90% c. 80% d. 75%
30. What is the median of the following numbers? 8,5,7,5,9,9,1,8,10,5 and 10.
a. 9 b. 8 c. 5 d. 7

31. A certain pole casts a shadow 24 feet long. At the same time another pole 3 feet high casts a
shadow 4 feet long. How high is the first pole, given that the heights and shadows are in
proportion?
a. 20 ft. b. 24 ft. c. 18 ft. d. 21 ft.

33. The perimeter of an isosceles triangle is 42 in. The 2 equal sides are each 3 times as long as
the third side. What are the lengths of the three sides?
a. 18, 21, 3 c. 21, 21, 21
b. 6, 6, 8 d. 18, 18, 6

36. If a certain job can be finished by 18 workers in 26 days, how many workers are needed to
finish the job in 12 days?
a. 45 b. 39 c. 24 d. 30

38. A box is 12 inches wide, 16 inches long, and 6 inches high. How many square inches of paper
would be needed to cover it on all sides?
a. 192 b. 720 c. 360 d. 900

39. P 8,000.00 is invested at 12% simple interest. What is the amount at the end of 2 years?
a. P 9,600 b. P 8,960 c. P 8,240 d. P9,920

Answer Key
1. B
2. B
3. D
4.B
5.D
6.A
8. C
9. B
10. C
11. C
12. C
13. A
14. D
15. C
16. B
17. A
18. B
19. B
20. D
23. B
24. D
25. C
26. B
28. C
29. D
30. B
31. C
33. D
36. B
38. A
39. D
Practice Test 2 in LET Physical Science

1. The radius of the path of an object in uniform circular motion is doubled. The centripetal force
needed if its speed remains the same is ________.
A. Four times as before
B. Half as great as before
C. Twice as much
D. Three times as much

2. Car A has a mass of 1000kg and a speed of 60km/hr. car B has amass of 2000kg and a speed
of 30km/hr. compare the kinetic energy foe car A with that of car B.
A. Half as much
B. Four times as much
C. Equal
D. Twice as much

3. What would be the horizontal line in a position-time graph means?


A. Changing position, constant velocity
B. Constant position, constant velocity
C. Changing position, increase velocity
D. Constant position, zero velocity

4. What is the cars acceleration if ten seconds after the starting from rest; it is moving at 40m/s?
A. 4.0m/s2
B. 10m/s2
C. 2.5m/s2
D. 0.25m/s2

5. A heavy object and a light object are released from the rest at the same height and time in a
vacuum. As they fall, they have equal ________.
A. Weights
B. Moments
C. Acceleration
D. Energies

6. Airplanes are designed with pointed nose and thin tails in order to__________.
A. Produce fluid friction
B. Reduce drag
C. Increase the lift of the wing
D. Lessen the forward thrust

7. A stone was thrown upward from the roof, at the same time that an identical stone was
dropped from there. The two stones will ________.
A. Have the same velocity upon reaching the ground
B. Take the same time to reach the ground
C. Have they same acceleration when they reach the ground
D. Reach the ground at the same time

8. In a pair of scissors, the fulcrum is located at the ________.


A. Handle
B. Screw
C. Tip of the blades
D. Sharp edges

9. What kind of energy is possessed by water falling from a dam?


A. Potential
B. Kinetic
C. Accelerating
D. Interacting

10. If an object were equipped with a speedometer and allowed to fall freely on a planet where the
acceleration due to gravity is 20m/s2, by, how much will the reading on the speedometer
increase each second?
A. 30m/s
B. 20m/s
C. 40m/s
D. 10m/s

11. According to the kinetic molecular theory, liquids are similar in solids in that _______.
A. Their molecules are arrange in a regular manner
B. The motion of their particles changes with temperature
C. The motion of their particles cannot be determined
D. There is very little space in their molecules

12. When the solid block of materials is cut in half, its density is ________.
A. Unchanged
B. Halves
C. Doubled
D. Tripled

13. A ball is launched at an angle of 60 degrees from the horizontal. What do you call the horizontal
distance traveled by the ball?
A. Trajectory
B. Projectile
C. Range
D. Hyperbola
14. Which of the following is TRUE about an object thrown horizontally?
A. The vertical distance traveled increases uniformly
B. The horizontal motion is uniformly accelerated
C. The horizontal component of velocity increases
D. The velocity of the subject increases

15. In the absence of the air resistance, a projectile has a maximum range when thrown at an
angle of __________.
A. 90 degrees
B. 60 degrees
C. 30 degrees
D. 45 degrees

16. A 400 N woman stands on top of a very tall ladder so she is one earth radius above the earths
surface. How much does she weights?
A. 100N
B. Zero
C. 200N
D. 400N

17. What must be the minimum length of the plane mirror in order for you to see a full view of
yourself?
A. Your full height
B. One-fourth
C. One-half
D. Three-fourths

18. A rifle recoils from firing a bullet. The speed of the recoil is slow because ___________.
A. The force against the rifle is less
B. The rifle has more mass than the bullet
C. The impulse on the rifle is less
D. The momentum of the rifle is unchanged.

19. Which statements differentiate weight for mass?


A. Weight is distance from a reference point
B. Weight has volume, mass has none
C. Weight is a full of gravity, mass is space occupied
D. Weight is a force while mass is amount of matter

20. Who established the relationship between the volume of a gas and the pressure when the
temperature is constant?
A. Blaise Pascal
B. Robert Boyle
C. Lavoisier
D. Archimedes

ANSWER KEY
1. B
2. D
3. D
4. A
5. C
6. B
7. C
8. B
9. B
10. B
11. B
12. A
13. C
14. D
15. D
16. A
17. C
18. B
19. D
20. B
Practice Test in LET MAPEH

1. To enjoy satisfying life, you need to maintain a healthy life style not only in food intake but
also activities that can develop endurance. What particular activities will do?
A. Practice yoga
B. Brisk walking and jogging
C. Fast running
D. Regular exercise

2. Cardiovascular endurance is necessary for you to become sprinter. Which activity promotes the
development of the component?
A. 3 to 5 k run regularly
B. Continuous running
C. Brisk walking
D. Stationary movement

3. Cardiovascular fitness, body composition, flexibility, muscular strength and endurance are the
specific test for one component of physical fitness. What component is being described?
A. Skill related component
B. Socially related
C. Health related component
D. Emotionally related

4. A related component that assure stability and maintenance of an upright posture while still or
moving and doing some routine activities.
A. Agility
B. Power
C. Reaction time
D. Balance

5. Ply metric training requires quick movement from one marker to another. What physical
components of a football player can be enhancing with this movement?
A. Strength
B. Speed
C. Agility
D. Flexibility

6. In an organization, which of the following is involved in the recruiting, subdividing, grouping


and coordinating the development of the various kinds of human resources?
A. Intramural
B. Inter-district
C. Inter-school
D. Division meet
7. It is based on the universal principle Sports of All wherein the DepEd professionalize sports
management in every school. Which sport management features physical activities such as
sports, dance contest, cheering squad and Mr. and Miss within the school only?
A. Intramural
B. Inter-district
C. Inter-school
D. Division meet

8. In sports elimination, if a team loses once, it is eliminated and only the winning team will
advance to the next game. What kind of sports elimination is being described?
A. Single round robin
B. Single elimination
C. Double elimination
D. Double elimination

9. In the double round robin type of tournament, what is the formula if the number of team is 7
and the number of game is 12?
A. N 1
B. N 1 x 2
C. N (-N 1)2
D. N (N 1)

10. In basketball, a foul is committed against the player who is able to make a goal in spite of the
foul. Does the shot counted?
A. Yes, a free throw is awarded
B. No, a free throw is awarded
C. No, a jump ball is called
D. Yes, the other team will take the ball

11. It is 3 seconds time, team A is behind with 2 points with ball possession. Player of team A who
is about to inbound the ball saw team mate near the opponents basket free and alone, in order
that the receiver be able to shoot the ball within the remaining time, what is the pass to be
execute?
A. Overhead pass
B. Chest pass
C. Baseball past
D. Bounce past

12. Stepping into the pass is the most significant factor to stress when executing a pass. What is
the most evident action to consider when making follow-through on the chest pass?
A. Arms are fully extended
B. Wrist are fully extend
C. Palms are facing downwards
D. Knees are straight

13. Basketball was first played in Springfield, Massachusetts in 1891. Who created the game?
A. Abner Doubleday
B. James Naismith
C. William Morgan
D. Kareem Abdul Jabbar

14. Who is considered as the fastest player in basketball and has the control of the ball and also
make it sure the ball gets to the right player at the right time?
A. Center
B. Power forward
C. Point guard
D. Shooting guard
15. Volleyball was first played in Tokyo, Olympics, 1964. It was created by William G. Morgan and
named it Mintonette in 1895. Who introduce volleyball in the Philippines in 1910?
A. Dr. A. F. Haistead
B. Elwood S. Brown
C. Isaac Spratt
D. Richard Cavill

16. One important skill in volleyball is service. How do you a serve wherein the ball is held at the
left hand and straight forward swing with the right hand, hitting the ball with an open hand or
fist, either facing forward of sideward serve?
A. Overhand serve
B. Underhand serve
C. Spike
D. Blocking

17. The player is hitting a forearm pass (underhand pass) and contacts the ball on thumbs. What
will probably happen to the ball?
A. The hit will be illegal
B. The control of the ball will be lost
C. The ball will be directed to the intended location
D. The ball might get out of the court

18. Total fitness is necessary in all players. Designated skills must be done and executed in
accordance to rule of the game. What characteristic is most advantageous for the setter?
A. Strong fingers
B. Flexible wrist
C. Good vision
D. Relax shoulder

19. The referees hand signals in case of faults or situation in finals. What is the situation to refer to
if the referees raise the forearms front and back and twist them around the body?
A. Substitution
B. Time-out
C. Change court
D. Blocking fault

20. What is the fault committed if the referees raise eight fingers, spread open?
A. Ball in
B. Ball out
C. Disqualification
D. Delay in service

ANSWER KEY
1. B
2. A
3. C
4. D
5. C
6. B
7. A
8. B
9. D
10. A
11. A
12. C
13. B
14. C
15. B
16. B
17. B
18. C
19. C
20. D
Practice Test in LET Professional Education-Instructional Materials

1. ________ is an example of learning task with direct authenticity.


A. Simulation
B. Role playing
C. Small group discussion
D. Filling in the blank application form

2. ________ is the process of organizing materials to make it clear for the learners.
A. Structuring
B. Elaborating
C. Sequencing
D. Balancing

3. Instructional materials must be connected with __________.


A. The course guide
B. The curriculum
C. The teaching technique
D. The level of students

4. Which is not a reason why it is not important for a teacher to know how to evaluate
instructional materials?
A. Writings ones material is time consuming
B. Teachers area able to identify learners need
C. Teachers area able to use the material properly and effectively
D. Not all books are error-free

5. It is important to _______ to ensure the relevance of instructional materials to the curriculum.


A. Base them on a textbook used
B. Include test evaluation
C. Consider the grade/year level
D. Have a list of goals and objectives

6. Under which phase/s of curriculum development do setting goals and objectives falls?
A. Planning
B. Implementation
C. Implementation and evaluation
D. Planning and implementation

7. Which is not a teachers personality factor that may influence the production of instructional
materials?
A. Financial status
B. Philosophy in life
C. Teaching style
D. Beliefs and values
8. A teacher is doing _________ when he studies skills covered, and activities presented in a
book.
A. Internal evaluation
B. Content evaluation
C. External evaluation
D. Physical evaluation

9. Which is not considered in designing instructional materials?


A. Socio-economic environment
B. Teachers creativity
C. Students and their schema
D. Teachers pedagogical principles

10. Materials are considered _______ if they are brought from the real world to the classroom to
expose learners to language in real use.
A. Authentic
B. Graded
C. Suitable
D. Appropriate

11. _________ Materials provides learners with communicative samples to different context.
A. Learner-authentic
B. Graded text
C. Learner centered
D. Textually authentic

12. Which is not under external evaluation?


A. Blurbs
B. Activities
C. Table of contents
D. Introduction

13. The communicative approach emphasizes _________.


A. Students interaction in real life situations
B. Students sharing of personal experiences
C. Listening and speaking
D. Correct pronunciation

14. ________ is an example of sequencing materials.


A. Whole to parts
B. By stages
C. Top to bottom
D. Bottom to top

15. What must be included in instructional materials?


A. Teaching strategies
B. Learning texts and tasks
C. Reference material
D. Material instruction

16. Which is correct about textually authentic materials?


A. Improve learners vocabulary
B. Promote interest in language learning
C. Interactive in character
D. Not written for teaching

17. This principle in preparing for an instructional material provides continues and cumulative
learning.
A. Pacing
B. Sequencing
C. Balancing
D. Evaluating

18. Preparation for instructional material falls under what phase/s of curriculum development?
A. Planning and implementation
B. Implementation and evaluation
C. Planning
D. Evaluation

19. _________ is an instructional material that a student can do without any help from the teacher.
A. Pictograph
B. Word paragraph
C. Module
D. Video tape

20. It is needed to make a list of what learners ________ in preparing the aims, goals and
objectives of the curriculum.
A. Do in make-believe situation
B. Should do as citizens in a democratic country
C. Are expected to do in school
D. Are required to do in real world

ANSWER KEY
1. D
2. A
3. D
4. A
5. D
6. D
7. A
8. B
9. A
10. A
11. D
12. C
13. A
14. B
15. B
16. D
17. A
18. A
19. C
20. D
Practice Test in LET Professional Education-Principles and Strategies in Teaching

1. The _________ method is used when the learners are made to observe things in a certain
place like the market.
1) Participatory
2) Case study
3) Simulation
4) Field trip

2. The recommended method to use if the teachers wishes each learner to concentrate in
learning a topic to his skills are properly assessed is.
1) Semantic webbing
2) Independent study
3) Role playing
4) Field trip

3. The ________ method is observed if we wish the pupil learn from real life situation dilemmas.
1) Situation
2) Lecture
3) Textbooks
4) Observation

4. The ________ method is used to make the learners study in detail a specific thing, person or
place not known to them
1) Case study
2) Participation
3) Project
4) Field trip

5. Which of the following method will you used to verify a certain findings and to make the
learners handle apparatus properly?
1) Textbook method
2) Laboratory method
3) Field trip method
4) Project method

6. The ________ method is utilized if the learners are trained to do creative products.
1) Project
2) Case study
3) Field trip
4) Simulation

7. If the material is dangerous for the learners to handle, which of the following method will you
use?
1) Textbook
2) Group discussions
3) Lecture-demonstration
4) Eclectic

8. Which method is used to develop scientific inquiry among the learners?


1) Project
2) Case study
3) Problem solving
4) Simulation
5)
9. The ________ method is used to find out the learners knowledge about a certain topic
assigned to them.
1) Independent study
2) Textbook
3) Lecture
4) Question and answer

10. The ________ approach is utilized when the learners are trained to ask intelligent question.
1) Process
2) Discovery
3) Inquiry
4) Value certification

11. If you wish to relate a subject matter to one of the four principles of learning, the ________
approach should be used.
1) Multidisciplinary
2) Interdisciplinary
3) Conceptual
4) Integration

12. The ________ approach is used if the teacher wishes to solve a problem being met in the
school.
1) Mastery
2) Integration
3) Action learning
4) Value clarification

13. You wish to make the learners learn or internalized fully a subject matter to be taught to them.
Which of the following will you need?
1) Integrated
2) Multi-media
3) Master
4) Multi-disciplinary

14. Which of the following approaches will be used if you wish to relate a particular subject to all
disciplines of learning?
1) Multidisciplinary
2) Interdisciplinary
3) Value clarification
4) Integration

15. To enable the learners to learn by their own pace of growth, _________ approach is used.
1) Inquiry
2) Discovery
3) Mastery
4) Modular

16. The ______ approach is observed when the learners want to meet the criterion level of success
act set by the teachers.
1) Mastery learning
2) Interdisciplinary
3) Conceptual
4) Modular

17. Which approach is used to emphasize the skills in informing conclusions?


1) Multidisciplinary
2) Interdisciplinary
3) Conceptual
4) Inquiry

18. Which of the following approaches is used to include issues confronting the societies?
1) Integration
2) Interdisciplinary
3) Conceptual
4) Inquiry

19. The _______ approach is used to make the learners enunciate their feelings or attitudes about
certain issues.
1) Value clarification
2) Mastery
3) Integration
4) Interdisciplinary

20. The _________ method is used if the learners are to use their senses effectively.
1) Textbook
2) Lecture demonstration
3) Observation
4) Independent study

ANSWER KEY
1. 4
2. 1
3. 4
4. 1
5. 2
6. 1
7. 3
8. 3
9. 4
10. 3
11. 4
12. 3
13. 3
14. 1
15. 2
16. 1
17. 3
18. 2
19. 1
20. 2
PRACTICE TEST IN MATH LET

PRACTICE TEST IN MATH LET

1. Which number is wrong in the series? 2, 4, 8, 14, 32

A. 4 B. 8 C. 14 D. 2

2. In a class of 10 boys and girls, the boys average score on the final exam was 80
and the girls average score was 90. What was the average score for the whole
class?

A. 85 B. 86 C. 87 D. 84

3. What is the average of 1/2, 1/4, and 1/3 ?

A. 13/24 B. 13/29 C. 13/27 D. 13/36

4. If the price of round-trip fare to Malolos, Bulacan is P 285, how much will it cost a
family of four if the husband pays full fare, the wife pays 2/3 of full fare, and the
two children each pay 1/2 of the regular price?

A. P 475.00 B. P 570.00 C. P 760.00 D. P 1,140.00

5. What is the missing term in the series? 2, 5, 11, 23, 47, __.

A. 115 B. 95 C. 105 D. 125

6. A store owner makes a 25% profit by selling an item for P 800? How much is his
profit?

A. P 1,400.00 B. P 200.00 C. P 250.00 D. P 160.00


7. Find the GCF of 18 and 36.

A. 2 B. 6 C. 18 D. 9

8. Neneth became heavier by 23 pounds due to increased appetite.She weighs 175


now.

What was her original weight?

A. 199 pounds B. 198 pounds C. 152 pounds D. 153 pounds

9. Write the expression 3x2 + x in factored form

A. 3x2 ( 1 + 1) B. 3x2 ( 1 + x ) C. x( 3x + 1) D. 3(x2 + 1 )

10. Which of the following is the smallest prime number greater than 200?

A. 214 B. 205 C. 201 D. 211

11. If a car travels 96 miles on 8 liters of gas, how far can the car travel on a full
tank of gas that holds 20 liters?

A. 235 B. 245 C. 230 D. 240

12. 4 - 2 = ________
A. 2 4/8 B. 1 4/8 C. 1 7/8 D. 2 7/8

13. A pillow measures 54 centimeters by 621 centimeters. How much braid is


needed to go around the pillow?

A. 2311 B. 2320 C. 1350 D. 2330

14. Which of these is NOT divisible by 11?

A. 484 B. 333,333 C. 913 D.


3,333,333

15. In a group of 250 students, 40 are seniors, what percentage of the group is
seniors?

A. 30 B. 16 C. 10 D. 40
16. If x- 6 > 24, then
A. x > 6 B. x > 30 C. x > 4 D. x >
24

17. Factor 65y3 - 35y2 + 15y


A. 5y ( 13y2 7y + 3 ) C. 5y3 (13 - 7y + 3)

B. 5 (13y3 7y + 3) D. 5( 13 - 7y2 + 3y )

18. Which of the following is/are TRUE?

I. x3 - y3 = ( x y )( x2 + y2 )
III. x2 + y2 = ( x + y )( x + y )

II. ( 5x 2y )( 5x 2y ) = 25x2 20xy + 4y2

A. I only B. I and III C. II only D. I


and II

19. Danilo has 5 blue T-shirts and 7 orange T-shirts. If he picks one T-shirt at
random, what is the probability that it will NOT be blue?
A. 1/2 B. 1/6 C. 7/12 D.
5/12

20. The Family Foods Corporation is redesigning its bran flakes box as shown below:

What is the capacity of the box in cubic inches?

A. 230 cubic in. B. 460 cubic in. C. 115 cubic in. D. 92 cubic in.

21. A 25- hectare field yields 350 cavans of palay. At this rate, how many cavans
will a 60-hectare field yield?
A. 830 B. 840 C. 850 D. 860

22. If y = 2x 5, what is the value of y when x= 3?


A. 1 B. 3 C. 1 D. 0

Answer key:

1. C

2. A

3. D

4. C

5. B

6. B

7. C

8. C

9. C

10. D

11. D

12. C

13. C

14. D

15. B

16. B

17. A

18. C

19. C

20. A

21. B

22. C
Practice Test in Professional Education-LET

1. You are very much interested in a quality professional development program for teachers. What
characteristic should you look for?

A. Prescribe by top educational teachers

B. Dependent on the availability of funds

C. Required for renewal of professional license

D. Responsive to identified teachers needs.

2. To ensure high standards of teachers personal and professional development, which of the
following measures must be implemented?

I. A school head plans the professional development of his/her teachers.

II. Every teacher formulates his/her own professional development plan.

III. The implementation of what is learned in a training must be monitored.

A. I only

B. I and III

C. II and III

D. II only

3. As a community leader, which of the following should a teacher NOT do?

A. Support effort of the community to improve their status in life.

B. Make herself aloof to ensure that her decisions will not be influenced by the community politics.
C.Solicit donation from philanthropists in the community.

D.Play an active part in the activities of the community.

4. In a highly pluralistic society, what type of learning environment is the responsibility of the
teacher?

I. Safe

II. Gender-biased

III. Secure

A. I and II

B. I, II and III

C. II only

D. I and III

5. A teacher is said to be trustee of the cultural and educational heritage of the nation and is
under obligation to transmit to learners such heritage. Which practice makes the teacher fulfill
such obligation?

A. Use interactive teaching strategies.

B. Use the latest educational technology.

C. Observe continuing professional education

D. As a class, study the life of Filipino heroes.

6. Which actions show that political factors affect schools as agents of change?

I. The strengthening of the teaching of English in Philippines school.

II. The introduction of mandated subjects such as Rizal in the curriculum

III. The practice of mainstreaming

IV. The turnover of day care centers from DSWD to DepEd supervision.

A. I and III

B. I and II

C. II and III
D. II and IV

7. For more efficient and effective management of school as agents of change, one proposal is
for the DepEd to cluster remote stand-alone schools under one lead school head. Which factor
has the strongest influence on this proposal?

A. Psychological

B. Historical

C. Geographical

D. Social

8. What does the acronym EFA imply for schools?

A.The acceptance of exclusive schools for boys and for girls.

B.The stress on the superiority of formal education over that of alternative learning system.

C.Practice of inclusive education

D.The concentration on formal education system

9. The wide acceptance of bottom up management style has influenced schools to practice
which management practice?

A.Exclusion of politicians from the pool of guest speakers during graduation exercises.

B.Prescription of what ought to be done from the Center Office.

C.Involvement of students, parents, teachers, and community in school planning

D.Allowing schools to do what they think is best

10. Large class size in congested cities is a common problem in our public schools. Which
measure/s have schools taken to offset the effects of large class?

I.The deployment of more teachers

II. The implementation of 1:1 pupil textbook ratio

III.The conduct of morning and afternoon sessions

A. I, II and III
B. I and II

C. III only

D. II only

11. The failure of independent study with most Filipino students may be attributed to _________
students

A. unpreparedness from schooling

B. ambivalence

C. high degree of independence

D. high degree of independence on authority

12. The shooting incidents in school campuses abroad have made school to rethink the curriculum.
Which is believed to counteract such incidents and so is being introduced in schools?

I.Inclusion of socio-emotional teaching

II.The emphasis on the concept of competition against self and not against others

III.Focus on academic achievement and productivity

A. I and III

B. II and III

C. I and II

D. I, II and III

13. Widespread abuse of Mother Earth prompted schools to teach sustainable development. Which
one does this prove about schools?

A.The curricula of schools are centered on Mother Earth.

B.Schools can easily integrate sustained development in their curriculum.

C.Sustained development cannot be effectively taught in the classroom.

D.Environment factors influence the school as an agent of change.

14.A father tells his daughter You are a woman. You are meant for the home and so for you,
going to school is not necessary. Is the father CORRECT?
A.It depends on the place when the daughter and the father live.

B.Yes, women are meant to be a mother only.

C.No. today women can take on the jobs of men.

D.No, there is gender equality in education.

15.Is there a legal basis for increasing the teachers starting salary to P18,000 a month?

A.No, it is a gift to teachers from Congress

B.Yes, R.A 7836

C.No, it is simply an act of benevolence from President GMA

D.Yes, the Phil. Constitution

16. As provided for the Education Act of 1982, how are the institutions of learning encourage to
set higher standards of equality over the minimum standards required for state recognition?

A. Granting of Special Permit

B. Academic freedom

C. Continuing Professional Education

D. Voluntary accreditation

17. Despite opposition from some school official, DepEd has continuously enforced the no
collection of fees policy during enrolment period in public schools. Is this policy in accordance
with EFA goals?

A.No, it violates the mandate of equality education

B.Yes, it somewhat eliminates gender disparities

C.Yes, it supports equitable access to basic education

D.No. it does not support parent of adult education

18. Specialization is knowing more and more about less and less. Hence, it is better to be a
generalist, claims Teacher F. Which Philosophy does Teacher F. subscribe to?

A. Existentialism
B. Perennialism

C. Essentialism

D. Progressivism

19. Mencius believed that all people are born good. This thought on the innate goodness of people
makes it easier to ________ our pupils.

A. teach

B. respect

C. like

D. motivate

20. The specialization requires of every professional teacher for him/her to be competent is in line
with which pillar of learning?

A. Learning to know

B. Learning to be

C. Learning to live together

D. Learning to do

ANSWER KEY

1. D

2. B

3. B

4. D

5. D

6. D

7. C

8. C

9. C

10. C
11. B

12. D

13. D

14. D

15. D

16. D

17. C

18. C

19. B

20. A
Practice Test in Social Science- Government and Constitution

1. All of the following constitute the meaning of political science except:


A. A basic knowledge and understanding of the state.
B. It is primarily concerned with the association of human beings into a political community.
C. Common knowledge every events taking place in the society.
D. It deals with the relationship among men and groups which are subject to the control by the stat
e.

2. It refers to the community of persons more or less numerous, permanently occupying a defi
nite portion of territory, having a government of their own to which the great body of inhabitan
ts render obedience, and enjoying freedom from external control.
A. Sovereignty
B. Nation
C. Citizenship
D. State

3. It refers to the agency through which the will of the state is formulated, expressed and carrie
d out.
A. Government
B. Sovereignty
C. Constitution
D. Laws

4. What are the four elements of state?


A.people, territory, sovereignty, government
B.people, constitution, territory, government
C. government, law, peace, territory
D. constitution, people, land, independence

5. What theory asserts that the early states must have been formed by deliberate and voluntary
compact among the people to form a society and organize government for their common good.
A. Necessity Theory
B. Divine Right Theory
C. Social Contact Theory
D. Social Compact Theory

6. Government exists and should continue to exist for the benefit of the people.
A. The statement is a general truth.
B. The statement is just an assumption.
C. The statement is a fallacy.
D. There is no basis for judgment.

7. What are the forms of government in which the political power is exercised by a few privilege
class.
A. Oligarchy and Aristocracy
B. Aristocracy and Monarchy
C. Theocracy and Fascism
D. Democracy and Tyranny

8. The precolonial Philippines has no established government. Its villages and settlements were
called barangays.
A. Only the first statement is true and correct.
B. Only the second statement is true and correct.
C. Both statements are true and correct
D. Both statements are untrue and incorrect.

9. There were four social classes of people in the precolonial barangays. They were the nobles,
freemen, serfs, and the slaves.
A. Only the first statement is true and correct.
B. Only the second statement is true and correct.
C. Both statements are true and correct.
D. Both statements are untrue and incorrect.

10. What are the two known written codes during the pre-Spanish era in the Philippines?
A. Maragtas and Kalantiaw Codes
B. Sumakwil and Sulayman Codes
C. Panay and Subanon Codes
D. Hammurabi and Ur Nammu Codes

11. Under the Spanish colonial government, who directly governed the Philippines?
A. The Governor-General
B. The Viceroy of Mexico
C. The Royal Audiencia
D. The King of Spain
12. What is the first city to be established in 1565 in the Philippines?
A. Manila
B. Davao
C. Cebu
D. Iloilo

13.The government which Spain established in the Philippines was defective. It was a governm
ent for the Spaniards and not for the Filipinos.
A. Only the first statement is true and correct.
B. Only the second statement is true and correct.
C. Both statements are true and correct.
D. Both statements are untrue and incorrect

14. What was the secret society founded in 1896 that precipitated the glorious revolution again
st the Spaniards.
A. The Katipunan
B. The Kalahi
C. The Biak na Bato Republic
D. The Ilustrado

15. Arranged the sequence of governments during the revolutionary era:


1. The Dictatorial Government
2. The Revolutionary Government
3. The Biak-na-Bato Republic
4. The First Philippine Republic
A. 2 3 1 4
B. 3 1 2 4
C. 4 1 3 2
D. 1 2 3 4

16. Arranged according to its establishment during the American Regime:


1. The Commonwealth Government
2. The Military Government
3. The Civil Government
A. 1 2 3
B. 2 3 1
C. 3 2 1
D. 2 1 3

17. What was the civil government established during the Japanese occupation of the Philippin
es?
A. The Japanese Imperial Government
B. The Philippine Republic
C. The Puppet Government of Japan
D. The Philippine Executive Commission

18. The Constitution used by the Philippine government from the commonwealth period until 1
973.
A. The Malolos Constitution
B. The Biak-na-Bato Constitution
C. The 1935 Constitution
D. The 1901 Constitution

19. What kind of government was installed under the 1973 Constitution under the Marcos regi
me?
A. Modified Presidential system
B. Modified Parliamentary system
C. Military system
D. Bicameral system

20. A de facto government acquires a de jure status when it gains wide acceptance from the p
eople and recognition from the community of nations.
A. The statement is true and valid.
B. The statement is an assumption.
C. The statement is a fallacy.
D. The statement is doubtful.

21. It is defined as written instrument by which the fundamental powers of the government are
established, limited and defined and by which these powers are distributed among the several
departments or branches for their and useful exercise for the benefit of the people.
A. Laws
B. Statutes
C. Constitution
D. Ordinances

22. There is no Constitution that is entirely written or unwritten.


A. The statement is true and correct.
B. The statement is incorrect.
C. The statement is partially correct.
D. There is no basis to conclude.

23. Requisites of a good written constitution.


A. Brief
B. Broad
C. Definite
D. All of the given options

24. Who has the authority to interpret the constitution?


A. Private individual
B. Courts
C. Legislative and Executive departments of the government
D. All of the given options

25. We, the sovereign Filipino people, imploring the aid of Almighty God , in order to build a ju
st and humane society and establish a government that shall embody our ideals and aspiration
s, promote our common good, conserve and develop our patrimony, and secure to ourselves an
d our posterity the blessings of independence and democracy under the rule of law and the regi
me of truth, justice, freedom, equality and peace, do ordain and promulgate this Constitution.

What part of Constitution is this?


A. General Provision
B. Amendments
C. Preamble
D. National Patrimony

Answer Key
1. C
2. D
3. A
4. A
5. C
6. A
7. A
8. C
9. C
10. A
11. D
12. C
13. C
14. A
15. B
16. B
17. D
18. C
19. A
20. C
21. A
22. D
23. D
24. C
25. C

You might also like